MSK III

अब Quizwiz के साथ अपने होमवर्क और परीक्षाओं को एस करें!

A 4-year-old male is brought to the pediatrician for a low-grade fever. His mother states that he has had a waxing and waning fever for the past 6 days with temperatures ranging from 99.8°F (37.7°C) to 101.0°F (38.3°C). She reports that he had a similar episode three months ago. She also reports symmetric joint swelling in the child's knees and wrists that has become increasingly noticeable over the past 8 weeks. He has not had a cough, difficulty breathing, or change in his bowel movements. The child was born at 40 weeks gestation. His height and weight are in the 45th and 40th percentiles, respectively. He takes no medications. His temperature is 100.1°F (37.8°C), blood pressure is 100/65 mmHg, pulse is 105/min, and respirations are 18/min. On examination, there is a non-pruritic, macular, salmon-colored truncal rash. Serological examination reveals the following: Serum: Rheumatoid factor: Negative Anti-nuclear antibody: Negative Anti-double stranded DNA: Negative Anti-SSA: Negative Anti-SSB: Negative Human leukocyte antigen B27: Positive Erythrocyte sedimentation rate: 30 mm/h This patient is most likely at increased risk of developing which of the following? A. Iridocyclitis B. Sacroiliitis C. Scoliosis D. Aortitis E. Dactylitis

A, Juvenile Idiopathic Arthritis The patient in this vignette presents with intermittent fever, polyarthritis, and rash suggestive of juvenile idiopathic arthritis (JIA). JIA is associated with iridocyclitis, which can lead to blindness if not identified and treated appropriately. JIA is a nonmigratory mono- or polyarthritis that occurs in prepubescent children. It is characterized by intermittent fever, joint pain (predominately in the knees, wrists, or hands), salmon-colored rash, and other extra-articular manifestations. An important complication of JIA is the increased risk of iridocyclitis, a form of uveitis. Patients with JIA therefore need to be screened regularly for vision changes or signs of uveitis to avoid potential blindness. Incorrect Answers: Answer 2: Sacroiliitis refers to inflammation of the sacroiliac joints. This condition is seen in patients with ankylosing spondylitis, a systemic autoimmune seronegative arthropathy which is associated with HLA-B27. Answer 3: Scoliosis refers to a spinal deformity characterized by abnormal curvature and rotation of the spine. Scoliosis is associated with some systemic disease including Ehlers-Danlos syndrome, Marfan syndrome, Charcot-Marie-Tooth disease, and cerebral palsy. Answer 4: Aortitis refers to inflammation of the aorta. It is seen in patients with ankylosing spondylitis, Takayasu arteritis, giant cell arteritis, polyarteritis nodosa, sarcoidosis, and others. Answer 5: Dactylitis refers to painful inflammation of the entire digit. Dactylitis can occur in seronegative arthropathies like psoriatic arthritis and ankylosing spondylitis or in vasoocclusive crises due to sickle-cell disease. Bullet Summary: Patients with juvenile idiopathic arthritis require regular ophthalmologic evaluation to reduce the risk of blindness secondary to uveitis.

A 55-year-old woman comes to the clinic complaining of joint pain and stiffness for the past year. The pain is mainly concentrated in her hands and is usually worse towards the late afternoon. It is described with a burning quality that surrounds the joint with some numbness and tingling. The stiffness is especially worse in the morning and lasts approximately for 15-20 minutes. Her past medical history is significant for recurrent gastric ulcers. She reports that her mother struggled with lupus and is concerned that she might have the same thing. She denies fever, rashes, ulcers, genitourinary symptoms, weight loss, or bowel changes. Physical examination is significant for mild tenderness at the distal interphalangeal joints bilaterally. What is the best initial medication to prescribe to this patient? A. Acetaminophen B. Aspirin C. Hydroxychloroquine D. Infliximab E. Methotrexate

A, Osteoarthritis This patient's bilateral joint stiffness and pain that involves the distal interphalangeal joints (DIP) are characteristic of osteoarthritis. The best initial pharmacotherapy is acetaminophen and NSAIDS; however, given this patient's past medical history, acetaminophen is the best option. Osteoarthritis (OA) is a degenerative joint disorder involving mechanical wear and tear of the joints that eventually leads to articular cartilage destruction. Patients often present with asymmetrical pain in weight-bearing joints that improves with rest and worsens with use (e.g., at the end of the day). OA commonly affects the knees, hips, interphalangeal joints, first carpometacarpal joints, and facet joints of the lower cervical and lumbar spine. When OA affects the hands, symptoms are usually bilateral and symmetrical with complaints of "aching" or stiffness in the morning that do not exceed more than 30 minutes. Management includes nonpharmacological therapy (e.g., exercise and weight loss), pharmacologic therapy (e.g., NSAIDs, acetaminophen, and other second-line treatments), and surgery. Incorrect Answers: Answer 2: Aspirin is not as appropriate as acetaminophen in this patient despite being one of the best initial therapy for osteoarthritis as her past medical history reveals gastric ulcers. Answer 3: Hydroxychloroquine is one the pharmacologic therapies for systemic lupus erythematosus (SLE). Although this patient reports a positive family history, her review of systems is negative for other symptoms of SLE (e.g., rash, ulcers, fever, or weight loss). Answer 4: Methotrexate is one of the pharmacologic therapies for rheumatoid arthritis (RA), which is a differential for any patients complaining of joint pain. Although this patient's presentation (symmetric joint pain and stiffness involving the hands that are worse in the morning) is like that of RA, it can be differentiated via the duration of stiffness (< 30 minutes) and joint involvement (DIP). Answer 5: Infliximab is a TNF-alpha inhibitor that can be used for the treatment of RA. This patient's signs and symptoms are more in line with OA. Bullet Summary: The best initial pharmacologic treatment for osteoarthritis is NSAIDs or acetaminophen.

A 69-year-old man presents to his primary care physician for pain when he walks. He states that the pain is the worst in his left great toe but is also present in his hips and knees. He says that his symptoms are worse with activity and tend to improve with rest. His symptoms have progressively worsened over the past several years. He has a past medical history of obesity, type II diabetes mellitus, smoking, and hypertension. He drinks roughly ten beers per day. His current medications include metformin, insulin, lisinopril, and hydrochlorothiazide. The patient has a recent travel history to Bangkok where he admits to having unprotected sex. On physical exam, examination of the lower extremity results in pain. There is crepitus of the patient's hip when his thigh is flexed and extended. Which of the following is the most likely diagnosis? A. Osteoarthritis B. Rheumatoid arthritis C. Infectious arthritis D. Gout E. Pseudogout

A, Osteoarthritis This patient is presenting with chronic pain in his joints that is worsened with activity and relieved by rest suggesting a diagnosis of osteoarthritis. Osteoarthritis occurs secondary to degeneration of the cartilage in the joint. Common locations for it to occur include, but are not limited to, the hip, knee, and great toe. Risk factors include trauma, overuse, increasing age, and obesity. Patients will typically complain of pain in the joint that is exacerbated by activity and relieved by rest. There is often pain and crepitus in the effected joint. Treatment involves weight loss, physical activity as tolerated, and NSAIDs as needed. Ultimately these patients may need surgery to manage their symptoms. Incorrect Answers: Answer 2: Rheumatoid arthritis presents with symmetric pain and stiffness in the joints that is worse in the morning and improves with activity. It typically occurs in female patients and can be associated with systemic symptoms. Anti-CCP is a specific test for making this diagnosis. Answer 3: Infectious arthritis is possible given this patient's history of unprotected sex. However, infectious arthritis tends to present with a hot, inflamed, and erythematous joint. The insidious onset of this patient's symptoms and the association with activity point more towards a diagnosis of osteoarthritis. Answer 4: Gout typically presents with severe pain and inflammation in the great toe. Though this patient is experiencing symptoms in his great toe, his symptoms are not described as sudden or episodic but rather are insidious. Of note this patient has risk factors for gout of obesity, alcohol consumption, and thiazide diuretic use. Answer 5: Pseudogout typically causes symptoms that are more insidious than gout and tend to affect the wrist, ankle, and knee. This patient's symptoms have worsened over years and are associated with pain with activity and crepitus suggesting a diagnosis of osteoarthritis. Bullet Summary: Osteoarthritis presents with pain in weight bearing joints that is typically worse with activity and relieved by rest.

A 72-year-old woman with a past medical history of diabetes and obesity presents to the clinic with joint pain. She has severe pain in both of her knees for which she has been taking ibuprofen daily. Her temperature is 98.2°F (36.8°C), blood pressure is 177/109 mmHg, pulse is 80/min, respirations are 15/min, and oxygen saturation is 98% on room air. Physical exam is notable for the finding in Figure A. Radiography of the knee shows the finding in Figure B. Which of the following is the most likely diagnosis? A. Osteoarthritis B. Reactive arthritis C. Rheumatoid arthritis D. Septic arthritis E. Systemic lupus erythematosus

A, Osteoarthritis This patient is presenting with osteoarthritis given her joint pain, narrowing of the joint space on radiography, and Heberden and Bouchard nodes on her hands. Osteoarthritis has an insidious onset. It most often affects the weight-bearing joints in the knees, hips, lumbar and cervical spine, as well as the hands. Radiographic changes include joint space narrowing, osteophytes, subchondral sclerosis, and subchondral cysts. Risk factors include age, obesity, excessive joint loading, trauma, and genetic predisposition. Diagnosis involves both radiographs and history and physical. The treatment of osteoarthritis is weight loss, strengthening of nearby/supporting muscles, NSAIDs, acetaminophen, and in refractory cases, joint replacement. Figure/Illustration A depicts Heberden (distal arrows) and Bouchard (proximal arrows) nodes which are found in osteoarthritis. Figure/Illustration B shows joint space narrowing in the knee (arrow). Incorrect Answers: Answer 2: Reactive arthritis is a seronegative spondyloarthropathy that manifests as an acute and asymmetric oligoarthritis that typically occurs 2-6 weeks after infection. Answer 3: Rheumatoid arthritis presents in women with symmetric joint pain that is usually worse in the morning and relieved with repeat joint use. It can be associated with other systemic manifestations like fever, fatigue, and malaise. Answer 4: Septic arthritis presents with a red, hot, and tender joint that must be managed with arthrocentesis, broad spectrum antibiotics, and surgical washout. Answer 5: Systemic lupus erythematosus commonly presents in a young woman with arthralgias (symmetric), malar/discoid rash, oral ulcers, serositis, and many other laboratory derangements. Bullet Summary: Osteoarthritis presents with a cold and painful joint (that is typically weight bearing) and is associated with a narrowing of the joint space on radiography.

A 69-year-old woman presents with pain in her hip and groin. She states that the pain is present in the morning, and by the end of the day it is nearly unbearable. Her past medical history is notable for a treated episode of acute renal failure, diabetes mellitus, obesity, and hypertension. Her current medications include losartan, metformin, insulin, and ibuprofen. The patient recently started taking high doses of vitamin D as she believes that it could help her symptoms. She also states that she recently fell off the treadmill while exercising at the gym. On physical exam you note an obese woman. There is pain, decreased range of motion, and crepitus on physical exam of her right hip. The patient points to the areas that cause her pain stating that it is mostly over the groin. The patient's skin turgor reveals tenting. Radiography is ordered. Which of the following is most likely to be found on radiography? A. Loss of joint space and osteophytes B. Hyperdense foci in the ureters C. Femoral neck fracture D. Posterior displacement of the femoral head E. Normal radiography

A, Osteoarthritis This patient is presenting with pain worse on exertion, decreased range of motion, and crepitus in her hip, suggesting a diagnosis of osteoarthritis. A loss of joint space and osteophytes (reactive bone formation) would be seen on radiography. Osteoarthritis presents due to wear and tear of the joint typically from repeat trauma in the setting of risk factors such as obesity. Pain can be located in the hip, and can be referred to the groin or knee. The classic presentation is pain that worsens with activity. Crepitus and decreased range of motion can be found on physical exam. Common radiological findings include a loss of joint space and osteophytes. Illustration A demonstrates a radiograph with decreased joint space and osteophytes in osteoarthritis. Incorrect Answers: Answer 2: Hyperdense foci in the ureters suggests a diagnosis of renal calculi. Though this patient's referred pain to the groin, history of renal failure, and possible hypercalcemia (due to increased vitamin D intake) could suggest this diagnosis, her other symptoms are more suggestive of osteoarthritis. Answer 3: Femoral neck fracture would present acutely with pain in the hip, and a shortened and externally rotated leg. This diagnosis is possible given the patient's recent fall; however, her other symptoms and less acute presentation suggest a diagnosis of osteoarthritis. Answer 4: Posterior displacement of the femoral head suggests a diagnosis of posterior hip dislocation. Posterior hip dislocation would present with a shortened and internally rotated leg and a more acute presentation. Answer 5: Normal radiography would be found with a minor sprain/muscular injury. This patient's history suggests a diagnosis of osteoarthritis. Bullet Summary: Osteoarthritis presents with joint pain worse with exertion and radiographical findings of a decreased joint space and osteophytes.

A 6-year-old boy with a history of multiple fractures is brought to his pediatrician by his mother, because she is concerned her child cannot hear her. On physical exam, kyphoscoliosis, poor dentition, bowing of long bones, and conductive hearing loss is noted. On genetic analysis, the patient has a COL1A1 gene mutation. The defect found in this patient is most likely associated with impaired formation of which of the following? A. Blood vessels B. Vitreous body of the eye C. Lens D. Cartilage E. Sclera

E, Osteogenesis Imperfecta / Brittle Bone Disease This patient's presentation is highly suggestive for osteogenesis imperfect (OI), a connective tissue disorder commonly caused by mutations in collagen type I synthesis. The sclera is derived from type I collagen, and this explains why patients with OI can have blue sclera. OI, or "brittle bone disease," is an inherited genetic connective tissue disorder with various phenotypic presentations. This disorder is most commonly autosomal dominant (AD), with mutations in the COL1A1 or COL1A2 genes (90% of OI cases). These mutations alter the structure of one of the two type I collagen alpha chains. Type I collagen plays a very important structural role in skin, sclera, ligament, tendon, and bone. Therefore, these patients present with blue sclera, hearing loss, poor dentition, and multiple fractures. Forlino et al. present a review article on OI. The AD form of this disease is caused by primary defects in type I collagen. The autosomal recessive (AR) form of this disease occurs at the level of post-translational modifications/folding, where there may be protein deficiencies impairing the interaction with type I procollagen. Harrington et al. present an update on the treatment and evaluation of OI. Physiotherapy, rehabilitation, orthopedic surgery, and consideration of bisphosphonate treatment are involved in the management of moderate to severe OI in children. When managing fractures, emphasize decreasing the degree and the amount of time the patient spends immobile. This decreases osteoporosis secondary to disuse. Bone mineral density can be increased with bisphosphonate therapy, thus decreasing bone pain and the incidence of fractures. Illustration A shows brownish and opalescent teeth seen in OI patients (dentinogenesis imperfecta). Illustration B shows blue sclera in OI caused by choroidal veins reflecting through the thinned sclera. Incorrect Answers: Answer 1: Blood vessels are composed of type III collagen. The vascular type of Ehler's Danlos is associated with alterations in type III collagen. Answers 2 and 5: The vitreous body of the eye as well as cartilage are composed of type II collagen. Answer 3: The lens is composed of type IV collagen. Alport syndrome is a genetic disease secondary to mutations involved in the production of type IV collagen. These patients present with kidney disease, misshapen lenses & altered retina color, and hearing loss.

A 77-year-old woman presents to her primary care physician for a yearly checkup. During the visit, she says that over the last year she has been noticing increasing pain in her hands and knees while trying to perform normal activities. Specifically, she says that the pain is not as bad in the mornings but gets worse over the course of the day. Physical exam reveals joint tenderness and decreased range of motion in her knees bilaterally. In addition, she also has bony swellings of the proximal interphalangeal joints of both hands. Which of the following pathological findings would most likely be seen in this patient's knees? A. Osteophytes and subchondral sclerosis B. Pannus formation from the synovium C. Pencil in cup erosive deformity D. Septic bacterial cultures in the joint E. Yellow subcutaneous nodular deposits

A, Osteoarthritis This patient who has joint pain that gets worse after use and swelling of the proximal interphalangeal joints of her hands most likely has osteoarthritis. This disease is characterized by osteophytes and subchondral sclerosis. Osteoarthritis is a degenerative disorder affecting the articular cartilage that is characterized by loss of chondrocytes and degradation of the extracellular matrix. This destruction leads to narrowing of the joint space, subchondral sclerosis, and development of osteophytes in affected joints. The most common joints affected are the proximal and distal interphalangeal joints of the hands as well as the hips and knees. Since the disease is not inflammatory, osteoarthritis will present with joint pain that is better in the morning and worse after use. In contrast, inflammatory disease such as rheumatoid arthritis will have morning stiffness because the immune system can lead to swelling and joint destruction overnight when there is no motion. Illustration A shows the characteristic joint changes that would be seen in osteoarthritis including joint space narrowing, subchondral sclerosis, and osteophyte formation. Incorrect Answers: Answer 2: Pannus formation from the synovium would be seen in rheumatoid arthritis; however, this disease would present with pain that is worse in the morning and gets better after use. Answer 3: Pencil in cup erosive deformity would be seen in psoriatic arthritis; however, this disease would present with dactylitis and psoriatic lesions that appear with pink plaques and silvery scale. Answer 4: Septic bacterial cultures in the joint would be seen in septic arthritis; however, septic arthritis would present over the course of days to weeks rather than months to years. Answer 5: Yellow subcutaneous nodular deposits would be seen in gout; however, this disease is typically monoarticular and involves the first metatarsophalangeal joint. Bullet Summary: Osteoarthritis presents with narrowing of the joint space, subchondral sclerosis, and development of osteophytes in affected joints.

A 13-year-old boy presents to the clinic with fractures of the tibia and fibula. He was playing with his younger brother when he was pushed down to the ground and felt a snap. On skeletal survey, he has multiple fractures of indeterminate ages. Ophthalmologic exam reveals the following (Figure A). A defect in which of the following is the likely cause of this patient's disease? A. Type I Collagen B. Alpha-1 Antitrypsin C. Type III Collagen D. Type IV Collagen E. Fibrillin

A, Osteogenesis Imperfecta / Brittle Bone Disease Osteogenesis imperfecta is a genetic bone disorder caused by a variety of gene defects. It most commonly follows an autosomal dominant inheritance pattern and is caused by abnormal type I collagen which results in multiple fractures with minimal trauma (which can even occur during the birth process). Blue sclerae (caused by translucency of connective tissue over the choroid), hearing loss (due to abnormalities in the middle ear bones), and dental imperfections (from lack of dentin) are also found in osteogenesis imperfecta (brittle bone disease). Figure A shows blue sclerae seen in osteogenesis imperfecta. Incorrect Answers: Answer 2: A deficiency in alpha-1 antitrypsin may cause emphysema. Answer 3: A deficiency in type III collagen is the most common cause of Ehlers-Danlos syndrome. Answer 4: A deficiency in type IV collagen causes Alport syndrome. Answer 5: A deficiency in fibrillin causes Marfan syndrome.

A 39-year-old woman with poorly controlled systemic lupus erythematosus (SLE) presents to her rheumatologist for a follow-up visit. She has had intermittent myalgias, arthralgias, fatigue, and skin rashes over the past 10 years that have acutely worsened over the past year. She works as a school teacher but reports that she has had to miss several days of work due to her symptoms. She has been on hydroxychloroquine for several years but does not feel that it is adequately controlling her symptoms. She does not smoke or drink alcohol. Her temperature is 99.2°F (37.3°C), blood pressure is 130/75 mmHg, pulse is 80/min, and respirations are 18/min. On exam, she is well-appearing and in no acute distress. She has erythematous, raised plaques with keratotic scaling and follicular plugging on her hands. The physician decides to trial the patient on low-dose prednisone to better control her symptoms. At 2 months of follow-up, she reports that her flares have subsided and she has been able to return to work full-time. If this patient continues this medication regimen, she will be at an elevated risk for developing which of the following? A. Femoral neck fracture B. Femoral shaft fracture C. Meniscal tear D. Osgood-Schlatter disease E. Proximal phalanx fracture

A, Osteoporosis The patient in this vignette presents with prolonged glucocorticoid use due to poorly controlled SLE. Chronic steroid use is a risk factor for osteoporosis which may present with fragility fractures of the hip. Osteoporosis is a condition characterized by decreased bone mass leading to an increased fracture risk. Fragility fractures - fractures that occur due to decreased bone mineral density in patients with osteoporosis - include fractures of the hip, distal radius, lumbar spine, and proximal humerus. There are numerous risk factors for osteoporosis including post-menopausal age, female sex, lifestyle factors (e.g., poor physical activity, smoking, alcohol use disorder, and insufficient calcium and vitamin D intake), medications (e.g., glucocorticoids, proton pump inhibitors, and lithium), and medical conditions (e.g., hyperparathyroidism, hyperthyroidism, and multiple myeloma). Incorrect Answers: Answer 2: Femoral shaft fractures often occur in the setting of high-energy trauma. Although patients with osteoporosis may be at an increased risk for all fractures, femoral shaft fractures are not typically considered "fragility fractures" due to the fact that high-energy trauma is required to fracture the femoral diaphysis. Answer 3: Meniscal tears often occur with a sudden twist of the knee. The meniscus is made of cartilage, not bone, and meniscal tears are not associated with osteoporosis. Answer 4: Osgood-Schlatter disease (also known as tibial tubercle apophysitis), refers to inflammation of the patellar ligament's insertion at the tibial tubercle. This often occurs in adolescents and is not associated with osteoporosis. Answer 5: Proximal phalanx fractures often occur in the setting of high-energy trauma to the finger. Although patients with osteoporosis may be at an increased risk for all fractures, proximal phalanx fractures are not typically considered "fragility fractures." Bullet Summary: Chronic steroid use is associated with an increased risk of developing osteoporosis which may lead to fragility fractures.

A 52-year-old man presents to his primary care physician for generalized pain. The patient states that he feels like his muscles and bones are in constant pain. This has persisted for the past several weeks, and his symptoms have not improved with use of ibuprofen or acetaminophen. The patient has a past medical history of alcohol abuse, repeat episodes of pancreatitis, constipation, and anxiety. He has a 22 pack-year smoking history. His temperature is 99.5°F (37.5°C), blood pressure is 140/95 mmHg, pulse is 70/min, respirations are 15/min, and oxygen saturation is 99% on room air. On physical exam, you note generalized tenderness/pain of the patient's extremities. Abdominal exam reveals normoactive bowel sounds and is non-tender. Dermatologic exam is unremarkable. Laboratory values are ordered as seen below. Hemoglobin: 12 g/dL Hematocrit: 36% Leukocyte count: 7,500/mm^3 with normal differential Platelet count: 147,000/mm^3 Serum: Na+: 138 mEq/L Cl-: 100 mEq/L K+: 4.2 mEq/L HCO3-: 24 mEq/L BUN: 20 mg/dL Glucose: 99 mg/dL Creatinine: 1.0 mg/dL Ca2+: 10.2 mg/dL Alkaline phosphatase: 252 U/L Lipase: 30 U/L AST: 12 U/L ALT: 10 U/L Which of the following is associated with this patient's condition? A. Hearing loss B. Bence Jones proteins C. Hypercalcemia D. Adenocarcinoma of the gallbladder E. Obstructive jaundice

A, Paget Disease of Bone (Osteitis Deformans) This patient is presenting with bone pain in the setting of an elevated alkaline phosphatase and a normal calcium, suggesting a diagnosis of Pagets disease of the bone. A possible complication of this pathology is hearing loss. Pagets disease of the bone presents with bone pain in the setting of an elevated alkaline phosphatase and a normal calcium/phosphate. The diagnosis can be supported with a high urine hydroxyproline. Radiography will reveal osteolytic/sclerotic lesions, and bone scan will demonstrate increased radiotracer uptake. Complications of Pagets disease of the bone include hearing loss (due to temporal bone enlargement and loss of bone mineral density in the cochlear capsule), headache, spinal stenosis, radiculopathy, leg bowing, fracture, osteosarcoma, and giant cell tumor of the bone. Incorrect Answers: Answer 2: Bence Jones proteins represents a diagnosis of multiple myeloma. Multiple myeloma presents with bone pain and an atypical fracture in an elderly patient. This is a less likely diagnosis in the setting of this patient's elevated alkaline phosphatase level and symptoms. The alkaline phosphatase level tends to be normal in multiple myeloma unless associated with a fracture. Answer 3: Hypercalcemia is a likely complication of multiple myeloma. In Pagets disease of the bone, calcium and phosphate levels are normal. Answer 4: Adenocarcinoma of the gallbladder is a complication of repeat episodes of acute cholecystitis. Though this patient's alkaline phosphatase level is elevated (which can be found in acute cholecystitis), his other symptoms and bone pain do not point toward this diagnosis. Answer 5: Obstructive jaundice is a possible complication of pancreatic cancer. Pancreatic cancer is a possible diagnosis in a patient with a history of alcoholism and pancreatitis. Obstruction of the bile duct with pancreatic cancer can result in an increased alkaline phosphatase as well. However, this patient's diffuse bone pain suggests Pagets disease of the bone as a more likely diagnosis. Bullet Summary: Paget's disease of the bone can cause hearing loss due to temporal bone enlargement/loss of bone mineral density in the cochlear capsule.

A 66-year-old woman presents to the emergency department with abdominal pain. Her symptoms began when she was eating dinner. She has a past medical history of obesity, constipation, intravenous drug use, and diabetes. The patient is instructed to be nil per os and is transferred to the surgical floor. Three days later she had a cholecystectomy and is recovering on the surgical floor. Her laboratory values are ordered as seen below. Hemoglobin: 11 g/dL Hematocrit: 33% Leukocyte count: 8,500/mm^3 with normal differential Platelet count: 197,000/mm^3 Serum: Na+: 139 mEq/L Cl-: 100 mEq/L K+: 4.3 mEq/L HCO3-: 25 mEq/L BUN: 20 mg/dL Glucose: 99 mg/dL Creatinine: 1.1 mg/dL Ca2+: 10.5 mg/dL Alkaline phosphatase: 533 U/L GGT: 50 U/L AST: 22 U/L ALT: 20 U/L The patient is currently asymptomatic and states that she feels well. Which of the following is associated with this patient's underlying condition? A. Blastic and lytic skeletal lesions B. Monoclonal plasma cell replication C. Reemergence of a hepatitis infection D. Repeat gastrointestinal tract obstruction E. Qualitative bone defect

A, Paget Disease of Bone (Osteitis Deformans) This patient is presenting with laboratory values suggestive of Paget disease of the bone which is associated with blastic and lytic skeletal lesions. Paget disease of the bone typically presents in elderly patients with an elevated calcium, elevated alkaline phosphatase, and a pathologic fracture. However, another very common presentation for Paget disease of the bone is an asymptomatic patient with an elevated alkaline phosphatase. To ensure that the alkaline phosphatase is coming from bone (since it can come from the GI tract and the skeleton), a gamma-glutamyltransferase (GGT) level can be used to differentiate the origin (as it will be elevated in liver or gallbladder pathology and normal in a skeletal pathology). Paget disease of the bone is associated with lytic and blastic lesions in the bone. Incorrect Answers: Answer 2: Monoclonal plasma cell replication describes multiple myeloma which would likely present with a very elevated calcium and pathological fractures or bone pain. Answer 3: Reemergence of a hepatitis infection is a possible cause of an elevated alkaline phosphatase level; however, one would expect other symptoms as well as an elevated GGT. Answer 4: Repeat gastrointestinal tract obstruction is unlikely as her gallbladder has been removed (though a retained stone could still cause these symptoms). Since she is asymptomatic this is possible but less likely. Answer 5: Qualitative bone defect describes osteomalacia which is possible in this patient; however, the extent to which her alkaline phosphatase is elevated is beyond what one would expect in osteomalacia. In addition, there is no reason to suspect osteomalacia in this patient other than her gender and age, which are risk factors. Bullet Summary: Paget disease of the bone can present in an asymptomatic patient with only an elevated alkaline phosphatase and a normal GGT level.

A 6-year-old boy is brought for primary care evaluation after he was adopted from an international orphanage. His new adoptive parents say that they are concerned he may have an infection because he has difficulty with walking and complains about bone and joint pain. Furthermore, they have noticed that he is very weak compared to other children his age. Physical exam reveals bowed legs as well as enlarged costochondral junctions that are palpable as bumps along the ribs. Which of the sets of laboratory findings shown in Figure A would most likely be seen in this patient? A. A B. B C. C D. D E. E

A, Rickets / Osteomalacia This patient with bone pain, weakness, bowed legs, and rachitic rosary most likely has rickets, which would present with decreased calcium, decreased phosphate, increased alkaline phosphatase, and increased parathyroid hormone. Rickets is caused by defective mineralization of cartilage in the epiphyseal growth plates in children. It can be caused by vitamin D deficiency, hypophosphatemia, renal tubular acidosis, or hypophosphatasia. It will present with bone and joint pain, muscle weakness, difficulty with walking, and fractures. Physical exam will show bowed legs, rachitic rosary line (thickenings of the costochondral cartilage), kyphosis, and poor growth. Finally, palpation of the bones will reveal tenderness. Incorrect Answers: Answer 2: Decreased calcium, increased phosphate, increased alkaline phosphatase, and increased parathyroid hormone would be seen in secondary hyperparathyroidism; however, this disease would not lead to bowed legs or a rachitic rosary. Answer 3: Increased calcium, increased phosphate, normal alkaline phosphatase, and decreased parathyroid hormone would be seen in hypervitaminosis D; however, this disease would present with fatigue, constipation, and weakness. Answer 4: Normal calcium, normal phosphate, increased alkaline phosphatase, and normal parathyroid hormone would be seen in Paget disease of the bone; however, this disease would present much later in life. Answer 5: Normal calcium, normal phosphate, normal alkaline phosphatase, and normal parathyroid hormone would be seen in osteoporosis; however, this disease would present with bone fragility later in life. Bullet Summary: Rickets will present with bone pain, weakness, bowed legs, and rachitic rosary in children.

A 12-year-old Caucasian male presents with his mother to the pediatrician's office complaining of right thigh pain. He reports that he has noticed slowly progressive pain and swelling over the distal aspect of his right thigh over the past two months. He denies any recent trauma to the area and his temperature is 100.9°F (38.3°C). On exam, there is swelling and tenderness overlying the distal right femoral diaphysis. Laboratory evaluation is notable for an elevated white blood cell (WBC) count and elevated erythrocyte sedimentation rate (ESR). A radiograph of the patient's right leg is shown. Biopsy of the lesion demonstrates sheets of monotonous small round blue cells with minimal cytoplasm. Which of the following genetic mutations is most likely associated with this patient's condition? A. t(8;14) B. t(11;22) C. APC inactivation D. TP53 inactivation E. RB1 inactivation

B, Ewing sarcoma In this pediatric patient with a small round blue cell tumor arising from the femoral diaphysis, the most likely diagnosis is Ewing sarcoma. Ewing sarcoma is caused by a t(11;22) translocation. Ewing sarcoma is a tumor of neuroectodermal origin that typically affects males who are less than 15 years old. It is the second most common bone tumor in children after osteosarcoma. It typically arises in the diaphysis of long bones (e.g., femur, tibia, and humerus) and presents with localized pain and fever. WBC and ESR may be elevated. On radiographic imaging, a large destructive lesion will be seen in the diaphysis of a long bone. Concentric layers of new bone growth can lead to an "onion skinning" or "sunburst" appearance on radiograph. Histologically, Ewing sarcoma consists of sheets of monotonous small round blue cells. Ewing sarcoma is a very aggressive tumor and often metastasizes early; however, it is often treatable and responsive to chemotherapy. Ewing sarcoma is caused by a t(11;22) translocation that causes a EWS-FLI fusion protein. Figure A is a radiograph showing an anterior-posterior view of the distal femoral diaphysis with periosteal reaction leading to the classic "onion skinning" appearance. Incorrect Answers: Answer 1: t(8;14) is associated with Burkitt lymphoma, a form of non-Hodgkin lymphoma seen in the pediatric population. It presents as a jaw mass in the endemic form and as an abdominal mass in the sporadic form. Answer 3: APC is a tumor suppressor gene and inactivation of APC is associated with familial adenomatous polyposis (FAP). FAP is associated with a dramatically increased risk of colon polyps and colorectal cancer. Answer 4: TP53 is a tumor suppressor gene and inactivation of TP53 is found in Li-Fraumeni syndrome. Li-Fraumeni syndrome is associated with an increased risk of multiple tumors including breast cancer, osteosarcoma, and soft tissue sarcoma. Answer 5: Inactivation of RB1 is found in retinoblastoma and also seen in some osteosarcomas. RB1 is a tumor suppressor gene so inactivation leads to decreased tumor suppressor activity. Bullet Summary: Ewing sarcoma is an anaplastic small blue cell tumor that typically affects children and arises in long bone diaphyses. It is caused by a t(11;22) translocation that creates an EWS-FLI fusion protein.

A 4-year-old boy presents to the pediatrician to establish care and to be evaluated before starting school. On presentation, he has a cast over his left arm and right leg and his parents say that he seems to break a bone every few months. His parents both travel extensively for work so he is often being taken care of by other relatives. Otherwise, he has met all of the developmental milestones though he appears to be shy and withdrawn. A radiograph is obtained showing multiple old fractures in his arms and legs bilaterally in various stages of healing. Physical exam reveals a blue tint to his sclera. Which of the following characteristics is associated with the most likely cause of this patient's symptoms? A. Child abuse B. Hearing loss C. Hypophosphatemia D. Joint laxity E. Precocious puberty

B, Osteogenesis Imperfecta / Brittle Bone Disease This patient with multiple fractures sustained at different times and blue sclera most likely has osteogenesis imperfecta, which is associated with hearing loss. Osteogenesis imperfecta is an autosomal dominant disease that is caused by a mutation in type I collagen production. This defect results in abnormally fragile bones that fracture easily starting in childhood. It is often confused for child abuse but can be distinguished by physical exam signs such as opalescent brown teeth, caused by abnormal dentin formation, as well as blue sclera, caused by exposure of choroidal veins under the thinned overlying tissue. Notably, osteogenesis imperfecta is associated with hearing loss as well as limb shortening and bowing. Incorrect Answers: Answer 1: Child abuse is an important factor to consider when evaluating any child with traumatic injuries; however, the associated findings of blue sclera point towards osteogenesis imperfecta as a more likely diagnosis. Answer 3: Hypophosphatemia is associated with rickets, which may also present with multiple fractures; however, this disease would present with bone pain, bowed legs, and costochondral thickenings rather than blue sclera. Answer 4: Joint laxity can be seen in Marfan syndrome and Ehler Danlos syndrome; however, these would present with either abnormally long limbs and cardiovascular defects or stretchy skin and frequent dislocations rather than blue sclera. Answer 5: Precocious puberty would be seen in McCune-Albright syndrome; however, this disease would present with cafe-au-lait spots, precocious puberty, and endocrine abnormalities rather than blue sclera. Bullet Summary: Osteogenesis imperfecta presents with multiple fractures starting in childhood and is associated with hearing loss.

A 60-year-old woman presents to her primary care physician with pain in her hands and knees. She reports to having this pain intermittently for at least 8 months and describes it being worse in the evening. On physical exam, there is bone deformity noted on the distal and proximal interphalangeal joints, swelling of the right knee, and tenderness upon palpation of the affected joints.

Osteoarthritis (snapshot)

A 65-year-old woman presents to her primary care physician for a wellness checkup. She states that she has felt well lately and has no concerns. The patient has a 12-pack-year smoking history and has 3 drinks per week. She is retired and lives at home with her husband. She had a normal colonoscopy 8 years ago and mammography 1 year ago. She can't recall when she last had a Pap smear and believes that it was when she was 62 years of age. Her temperature is 98.1°F (36.7°C), blood pressure is 137/78 mmHg, pulse is 80/min, respirations are 13/min, and oxygen saturation is 98% on room air. Physical exam is within normal limits. Which of the following is the best next step in management? A. Colonoscopy B. DEXA scan C. Mammogram D. No intervention needed E. Pap smear

B, Osteoporosis This female patient who is 65 years of age with risk factors for osteoporosis should be screened with a DEXA scan. Osteoporosis typically presents in elderly women, in particular those with risk factors which include smoking, a sedentary lifestyle, and poor dietary habits. Complications from osteoporosis include fractures, such as compression fractures of the spine or femoral neck fractures. Women 65 years of age or older should be screened with a DEXA scan to assess bone density. Osteoporosis can then be treated/mitigated with resistance training and vitamin D and calcium supplementation. Incorrect Answers: Answer 1: Colonoscopy should be performed every 10 years starting at 50 years of age and ending at 75 years of age. This patient had a normal colonoscopy 8 years ago. Answer 3: Mammograms should be performed every 1 to 2 years starting at 50 years of age and ending at 75 years of age. Though it has been 1 year, she could wait up to another year to receive her mammogram. Thus, a DEXA scan is the more appropriate answer. Answer 4: No intervention needed misses the valuable opportunity to assess this patient's bone density with a DEXA scan and goes against screening recommendations. Answer 5: Pap smears should be started at 21 years of age, repeated every 2 to 3 years, and can be stopped at 65 years of age. Bullet Summary: DEXA scans should be performed in women 65 years of age or older, in particular those with risk factors such as smoking or a sedentary lifestyle.

A 58-year-old woman presents to her primary care physician for a wellness checkup. She recently had a DEXA scan that placed her at 2 standard deviations below the mean for bone density. She is following up today to discuss her results. The patient has a past medical history of asthma, breast cancer, COPD, anxiety, irritable bowel syndrome, endometrial cancer, and depression. She is currently taking clonazepam, albuterol, and fluoxetine. Her temperature is 99.5°F (37.5°C), blood pressure is 127/68 mmHg, pulse is 90/min, respirations are 15/min, and oxygen saturation is 95% on room air. The patient is treated appropriately and sent home. She returns 1 month later for a follow up visit. She has been taking her medications as prescribed. She endorses episodes of feeling febrile/warm which resolve shortly thereafter. Otherwise she is doing well. Which of the following is true of the medication she was most likely started on? A. Estrogen receptor agonist in the uterus B. Estrogen receptor antagonist in the uterus C. Induces osteoclast apoptosis D. Mineral replacement E. Parathyroid hormone analogue

B, Osteoporosis This patient is being treated for osteoporosis and is now presenting with hot flashes suggesting treatment with raloxifene. Raloxifene is an estrogen receptor antagonist in the uterus. Osteoporosis occurs secondary to decreased mineralization of bone. It typically occurs in sedentary patients who lead unhealthy lifestyles (poor diet and smoking). Raloxifene is a potential treatment for osteoporosis. It is an estrogen receptor agonist in bone but an antagonist in the uterus and breast. Thus, it can safely be used in patients who have a history of gynecologic malignancy. Other side-effects of raloxifene include hot flashes and venous thromboembolism. Incorrect Answers: Answer 1: Estrogen receptor agonist in the uterus describes the mechanism of action of tamoxifen. Tamoxifen would be an inappropriate drug to start in this patient given her history of endometrial cancer as it is an estrogen receptor agonist in the uterus. Answer 3: Induces osteoclast apoptosis describes the mechanism of action of bisphosphonates which are the best initial therapy for osteoporosis. They do not cause hot flashes as a side effect. Answer 4: Mineral replacement describes giving the patient calcium (often with vitamin D) which would be an appropriate initial intervention. Calcium replacement does not cause hot flashes. Answer 5: Parathyroid hormone analogue describes teriparatide. This is not a first-line therapy and does not cause hot flashes. Bullet Summary: Raloxifene can be used as a second-line treatment in osteoporosis but has side-effects of hot flashes and venous thromboembolism.

A 55-year-old male presents with left hip pain and stiffness. Radiographs are shown in Figures A and B. Serum alkaline phosphatase levels are elevated. A biopsy of the left femur is performed and shown in Figure C. Which of the following cells are initially responsible for this condition? A. Osteoblasts B. Osteoclasts C. Neutrophils D. T-Cells E. Fibroblasts

B, Paget Disease of Bone The radiographs and histology are consistent with Paget disease, which is initially caused by increased osteoclastic bone resorption, and followed by an increase in bone formation through osteoblast activity. Paget disease of the bone is a disorder of bone remodeling characterized by rapid bone turnover. The excessive bone resorption and bone formation disrupt normal bone architecture and lead to various complications such as bone pain, osteoarthritis, pathological fracture, bone deformity, deafness, and nerve compression syndromes. Schneider et al. describe Paget disease, also known as osteitis deformans, as a nonmalignant disease. The origin of the disease remains unknown and many patients are frequently asymptomatic. However, symptomatic individuals most commonly present with pain over the affected bone. Diagnosis is primarily made by radiographs. Bisphosphonates are the most common treatment. Delmas et al. review the use of total alkaline phosphatase as a serum marker for monitoring the activity of Paget disease. The authors report that bone specific alkaline phosphatase has the greatest sensitivity in Paget disease. Figure A shows the skull with areas of lysis, areas of sclerosis ('cotton-wool spots'), and calvarial thickening. Figure B demonstrates coarsened trabeculae, cortical thickening, and enlargement of the left femur. Figure C is a histologic slide showing abnormal bone formation with a mosaic pattern of woven bone and irregular sections of thickened trabecular bone and numerous cement lines. Incorrect Answers: Answer 1: While osteoblasts are involved in the pathogenesis of Paget disease, the primary disturbance is an exaggeration of osteoclastic bone resorption. Answers 3-5: None of these cells are implicated in the primary disturbance causing Paget disease.

A 78 -year-old gentleman presents to his primary care physician with progressive hearing loss over the last year. The patient has also been complaining of increasing pain in his legs over a similar time period. A radiograph of his right knee is shown in Figure A. Which of the following laboratory abnormalities is most likely in this patient? A. Decreased sodium B. Elevated alkaline phosphatase C. Elevated creatinine kinase D. Decreased phosphate E. Decreased calcium

B, Paget Disease of Bone This patient is suffering from Paget's bone disease. Overactive osteoclasts and bone remodeling causes progressive hearing loss and leg pain. Paget's bone disease is caused by overactive osteoclasts and bone remodeling. Affected bone has a characteristic appearance on x-ray, with poorly defined sclerotic and lucent regions. The most common symptom is bone pain; however disease that effects the skull can compress nerves resulting in progressive neurologic deficits (i.e. hearing and vision loss). The majority of cases remain mild and go undiagnosed. The signature laboratory abnormality is elevated serum alkaline phosphate secondary to increased osteoclast activity, and bisphosphanates are the first line treatment. Schneider et al. review the diagnosis and management of Paget's bone disease. They describe the disorder as bone resorption, followed by accelerated and chaotic bone deposition. Diagnosis is often made by radiograph. Ralston and Layfield review the pathogenesis of Paget's bone disease. It is believed that a large number of cases are due to a mutation in the SQSTM1 gene that increases osteoclast activation. Environmental factors also appear to play a significant role, with paramyxovirus identified as a possible trigger. Figure A shows a right knee consistent with Paget's disease. There are lucent areas with expansion and thinned, intact cortices mixed with other areas more proximally having bone enlargement with cortical thickening. Illustration A is an x-ray of a patient with Paget's bone disease of the skull. Note the cotton wool appearance of the bone with numerous sclerotic and lucent regions. Incorrect Answers: Answer 1: Sodium status in these patients should remain within the normal range. Answer 3: Kidney function should not be effected by Paget's bone disease Answer 4,5: Serum calcium and phosphate should be normal in these patients.

A 3-year-old boy is accompanied by his mother to his pediatrician for an annual physical. His mother says that he has recently begun daycare, and has been playing well with the other children. However, a few weeks ago he developed a rash that "looks a lot like acne" on his knees and elbows (Figure A). His mother additionally reports that he is noticeably smaller than the other children at his daycare. Temperature is 99.0°F (37.2°C), blood pressure is 102/73 mmHg, pulse is 95/min and regular, and respirations are 20/min. Physical exam reveals a thin boy with a scaphoid abdomen. His height is less than the 3rd percentile for his age. Mild genu varum is present, along with the findings in Figure B. Which of the following best explains the etiology of this patient's condition? A. Congenital proximal renal tubular acidosis B. Chronic fluoride toxicity C. Decreased absorption of fat soluble vitamins D. Decreased activity of renal 25-hydroxyvitamin D3 1-alpha-hydroxylase E. Mutation in tissue nonspecific alkaline phosphatase (TNSALP) gene

C, Rickets / Osteomalacia This patient's presentation with short stature, genu varum, costochondral thickening, and dermatitis herpetiformis are consistent with a diagnosis of celiac disease with concomitant rickets secondary to malabsorption of vitamin D. Celiac disease is caused by gluten intolerance and leads to destruction of intestinal villi. It is associated with the dermatologic condition dermatitis herpetiformis and causes diarrhea and malabsorption of fat-soluble vitamins, among them, vitamin D. Vitamin D is essential for proper calcium absorption and bone mineralization, and low levels in children can lead to defective mineralization of epiphyseal cartilage, causing rickets. Children with rickets will present with bowing of the legs (genu varum), rachitic rosary lines (costochondral thickenings), kyphosis, and poor growth. Figure A demonstrates the cutaneous findings of dermatitis herpetiformis. Figure B demonstrates costochondral thickening (rachitic rosary). Incorrect Answers: Answer 1: Congenital proximal renal tubular acidosis (Fanconi syndrome) causes hypophosphatemia along with a metabolic acidosis that increases calcium loss. Hypophosphatemia itself can cause rickets; however, this patient bears stigmata of celiac disease, making vitamin D deficiency due to malabsorption more likely. Answer 2: Chronic fluoride toxicity can cause osteofluorosis. This leads to brittle bones that are prone to fracturing. Answer 3: Decreased activity of renal 25-hydroxyvitamin D3 1-alpha-hydroxylase can lead to vitamin D deficiency and therefore rickets. However, this mechanism is only seen in children with advanced renal disease. Answer 5: Mutation in tissue-nonspecific alkaline phosphatase (TNSALP) gene is the cause of the autosomal recessive disorder hypophosphatasia. This mutation leads to a buildup of pyrophosphate, which inhibits bone mineralization. However, this disorder is exceedingly rare, and in a patient that bears stigmata of Celiac disease malabsorption of vitamin D is a far more likely etiology. Bullet Summary Rickets is due to poor mineralization of the epiphyseal cartilage in children and has a variety of etiologies including dietary vitamin D deficiency, type II renal tubular acidosis, chronic kidney disease, and hypophosphatasia.

A 67-year-old male presents with left hip pain. Examination reveals mild effusions in both knees, with crepitus in both patellofemoral joints. He states his hearing has worsened recently and that he feels like his hats don't fit anymore. Bone scan reveals diffuse uptake in the calvarium, right proximal femur, and left ilium. Which of the following laboratory abnormalities would be expected in this patient? A. Decreased serum alkaline phosphatase B. Increased serum alkaline phosphatase C. Decreased serum parathyroid hormone D. Increased serum parathyroid hormone E. Increased serum calcium

B, Paget Disease of Bone (Osteitis Deformans) The patient in this vignette most likely has Paget's disease, classically presenting with hearing loss, increased skull circumference, and an elevated alkaline phosphatase. Paget's disease is a autosomal dominant disorder involving abnormal bone remodeling with increased osteoclastic bone resorption and increased osteoblastic bone formation. This condition classically involves the pelvis and skull, but may also involve the spine, femur, and tibia. There is a peak incidence in the 5th and 6th decade of life, and the disease is more common in males (male:female ratio is 8:1). Less than 1% will develop malignant Paget's sarcoma (secondary sarcoma), which carries a poor prognosis. Schneider et al. discuss Paget's disease of bone (also known as osteitis deformans). They report that the origin is unknown, and although frequently asymptomatic, the most common symptom is pain in the affected bones. Other symptoms involve neurologic, hearing, vision, and cardiac systems. Bolland and Cundy review recent research involving Paget's disease. They note that mutations in the SQSTM1 gene occur in 25-50% of familial Paget's. Genotype-phenotype relationships are present in families with specific mutations. Clinical trials have solidified the role of bisphosphonates in the management of Paget's by reducing pain, improving quality of life, reducing bone turnover, and healing lytic lesions. Illustration A demonstrates bone histology in Paget's disease. Note the irregular broad trabeculae with disorganized cement lines and numerous large osteoclasts with multiple nuclei per cell. Incorrect Answers: Answer 1: Serum alkaline phosphatase levels are increased in Paget's disease. Answer 3: Serum parathyroid hormone levels are normal in Paget's disease. Answer 4: Serum parathyroid hormone levels are normal in Paget's disease. Answer 5: Serum calcium levels are normal in Paget's disease.

A 70-year-old woman presents to the office for a yearly physical. She states she has recently started experiencing pain in her legs and her back. Last year, she experienced a fracture of her left arm while trying to lift groceries. The patient states that she does not consume any dairy and does not go outside often because of the pain in her legs and back. Of note, she takes carbamazepine for seizures. On exam, her vitals are within normal limits. You suspect the patient might have osteomalacia. Testing for which of the following is the next best step to confirm your suspicion? A. 7-dehydrocholesterol B. 25-hydroxyvitamin D C. 1,25-hydroxyvitamin D D. Pre-vitamin D3 E. Dietary vitamin D2

B, Rickets / Osteomalacia An elderly woman with bone pain and history of low dairy intake combined with lack of sunlight exposure is likely suffering from osteomalacia. The next best test to confirm the diagnosis is 25-hydroxyvitamin D. Vitamin D deficiency commonly presents as lower back pain, proximal muscle weakness, muscle aches, and bone pain elicited with pressure. To confirm the diagnosis, a 25-hydroxyvitamin D level should be obtained in patients with suspected vitamin D deficiency. Deficiency is defined as a serum 25-hydroxyvitamin D level of less than 20 ng per mL. Treatment of choice is repleting vitamin D stores with the goal in adults to decrease the risk of fractures and falls. Bordelon et al. discuss recognition and management of vitamin D deficiency in adults. Risk factors include age over 65, dark skin, poor diet (no dairy), insufficient sunlight exposure, medications that alter vitamin D metabolism (e.g. anticonvulsants), and sedentary lifestyle. They also state that previous studies with vitamin D dosage of 700 to 800 IU per day reduced the relative risk of hip fracture by 26 percent and the relative risk of nonvertebral fracture by 23 percent when compared with calcium supplementation alone or placebo. Moraes et al. report on the effect of vitamin D deficiency in critically ill patients. In their study, they report that vitamin D deficiency is an independent risk factor for increased mortality in this patient population. They show that mortality rates were higher among patients with vitamin D levels <12 ng/mL (32.3%) versus vitamin D levels >12 ng/mL (13.3%), with an adjusted relative risk of 2.2 (P<0.05). Illustration A shows the conversion of vitamin D to its active form, 1,25-hydroxyvitamin D. Incorrect Answers: Answer 1 and 4: 7-Dehydrocholesterol in the skin gets converted to pre-vitamin D3 and is therefore not an accurate representation of the patient's vitamin D stores. Answer 3: 1,25-Hydroxyvitamin D is the active form of vitamin D found in the human body. Its levels fluctuate with levels of parathyroid hormone and calcitonin therefore it is not the best representation of the patient's vitamin D stores. Answer 5: Dietary vitamin D2 and D3 levels change on a daily basis and are therefore not best representation of the patient's vitamin D stores.

A 6-year-old girl presents with bowed legs and loose joints. Examination of her chest is shown in Figure A. Her growth chart shows she is falling further behind in growth and is currently at the 45th percentile. Laboratory results reveal a low serum phosphorous, an elevated alkaline phosphatase and low serum calcium. Which of the following would you expect to see histologically? A. Thinned trabeculae B. Unmineralized osteoid matrix with widened osteoid seams C. Woven bone and broad trabeculae, disorganized cement lines in a mosaic pattern D. Islands of calcified cartilage within mature trabeculae E. Round plasma cells with an eccentric nucleus, prominent nucleolus, and clock face organization of chromatin

B, Rickets / Osteomalacia The vignette above describes a girl with rickets; most likely vitamin D resistant rickets, the most common form of rickets. Rickets is characterized by inadequate mineralization of the osteoid, due to inadequate calcium or phosphate. If this same pathology were seen in adults, it would be called osteomalacia. Vitamin D resistant or familial hypophosphatemic rickets follows an X-linked dominant inheritance pattern and is due to the inability of the renal tubules to absorb phosphate. The result is decreased serum phosphorous and increased alkaline phosphatase. This leads to the histological findings of increased unmineralized osteoid matrix with widened osteoid seams. Vitamin D deficient rickets manifests similarly, but is due to a nutritional deficiency of vitamin D and has become much more rare after the fortification of milk with vitamin D. Nield et al. group rickets into vitamin D dependent rickets (patients respond to supplemental vitamin D), and vitamin D resistant rickets. The latter category includes familial hypophosphatemic rickets (described in the question stem) and hereditary hypophosphatemic rickets with hypercalciuria, a rarer disease that is treated with phosphorous alone. Casey et al. describe how vitamin D deficiency can lead to rickets and growth retardation in infants, children and adolescents. Those in whom supplementation with vitamin D is recommended include infants who are solely breastfed and anyone drinking less than 1 L of vitamin D fortified milk. The goal of supplementation is reaching 400 IU of vitamin D daily. UV-B exposure also increases vitamin D levels, and decreased sun exposure or darker pigmentation can lead to an increased risk of vitamin D deficiency. Figure A illustrates rachitic rosary, an enlargement of the costal cartilage. This physical exam finding is associated with rickets. Incorrect Answers: Answer 1: Thinned trabeculae are often seen in conjunction with thinned cortices in osteoporosis. Answer 3: Woven bone with cement lines in a mosaic pattern is describing Paget's disease. This disease is usually seen in the 5th decade of life and is a result of abnormal bone remodeling. Answer 4: Islands of calcified cartilage within mature trabeculae is describing the histologic findings of osteopetrosis. This is a metabolic bone disease caused by defective osteoclastic resorption of immature bone. Answer 5: Round plasma cells with clock face organization of chromatin is describing multiple myeloma. This is a neoplastic proliferation of plasma cells that presents with skeletal lesions.

A 31-year-old man presents to his primary care provider with left knee pain. He noted a bulge on his left knee 6 months ago that has gradually grown in size. A radiograph of the left knee is shown demonstrating a large lytic tumor in the fibular head. The patient is diagnosed with a giant cell tumor.

Benign Bone Tumors (snapshot)

A 7-year-old girl presents to her primary care physician for a routine check-up. The physician allows the medical student to perform a physical examination. The medical student notes hearing impairment as well as the findings show in Figures A and B. Radiographs show indications of multiple old fractures of the humerus that have healed. After questioning the girl's parents, the medical student learns that in addition, the patient is extremely picky with her food and eats a diet consisting mainly of cereal and pasta. What is the most likely etiology of the patient's disease? A. Decreased bone mineral density B. Defective mineralization of cartilage C. Deficiency of type 1 collagen D. Dietary deficiency of ascorbic acid E. Non-accidental trauma

C, Rickets / Osteomalacia This young patient's presentation of blue sclera, hearing impairment, tooth abnormalities, and multiple fractures are characteristic of osteogenesis imperfecta, a genetic bone disorder caused by a deficiency of type 1 collagen. Osteogenesis imperfecta (OI) causes decreased synthesis of type 1 collagen due to a problem with glycosylation of pro-alpha chain hydroxylysine residues and the formation of procollagen (triple helix of 3 collagen α chains). OI is most commonly inherited in an autosomal dominant pattern and manifests clinically as multiple fractures with minimal trauma (e.g., during the birth process), blue sclera due to the reflection of the choroidal veins over the thin, collagen-deficient connective tissue of the sclera, discolored and brittle teeth, and hearing loss due to abnormally-formed ossicles. OI may be confused with non-accidental trauma, or child abuse, due to the presence of multiple fractures in a child. However, in a patient who exhibits several other manifestations that are characteristic of OI, non-accidental trauma is less likely to be the cause for the fractures. Figure A shows an eye with blue sclera in a patient with osteogenesis imperfecta. Figure B shows discolored, worn-out teeth in a patient with osteogenesis imperfecta. Incorrect Answers: Answer 1: Decreased bone mineral density is the cause of osteoporosis, a condition in which trabecular and cortical bone lose mass and interconnections despite normal bone mineralization and serum calcium and phosphate levels. Osteoporosis can also lead to fractures , particularly vertebral compression fractures, but is less likely in this young child with characteristics that are more congruent with OI. Answer 2: Defective mineralization of cartilage in the epiphyseal growth plates is the cause of rickets, a condition that causes bone abnormalities such as bow legs, bead-like costochondral junctions, and craniotabes (soft skull) in children. Answer 4: Dietary deficiency of ascorbic acid (vitamin C) is the cause of scurvy, a disease which presents with swollen gums, bruising, petechiae, hemarthrosis, anemia, poor wound healing, and perifollicular and subperiosteal hemorrhages. Answer 5: Non-accidental trauma should be considered in a child presenting with multiple, recurrent fractures. However, in this scenario, the child's multiple other clinical features suggest that OI is the more likely diagnosis. Bullet Summary: Osteogenesis imperfecta is due to defective type 1 collagen synthesis.

A 17-month-old girl was brought to the emergency department by her mom following a fall. The mom reports that the patient was playing in the playground when she tripped and fell onto the mulch. She had an uncomplicated birth history and has been meeting developmental goals except for language delay, for which she is to receive a hearing test for further evaluation next week. Physical exam reveals bruising along the left lateral thigh, knee, and elbow; all lab tests are within normal limits. Radiograph shows a fracture of the olecranon. Serum chemistry and liver panels were within normal limits. What is the most likely explanation for the patient's presentation? A. Accidental trauma B. Child abuse C. Defective type 1 collagen gene D. Low levels of phosphate E. Low levels of vitamin D

C, Osteogenesis Imperfecta / Brittle Bone Disease The patient is presenting with signs of osteogenesis imperfecta (OI), which includes bone fractures following minor trauma and hearing loss (i.e., language delay secondary to hearing loss). OI is due to a defective type 1 collagen gene. OI is an inherited connective tissue disorder that can vary in its phenotypic presentations. Affected patients may suffer multiple fractures with minimal to no trauma. The disease is most commonly caused by mutations in the genes that encode for the alpha-1 and alpha-2 chains of type 1 collagen. Clinical features include atypical fractures, scoliosis, blue sclerae, hearing loss, easy bruising, and opalescent teeth. Diagnosis is based on clinical findings and a positive family history. Incorrect Answers: Answer 1: Accidental trauma is possible but falling in the playground onto mulch is generally not enough force to cause bone fractures. Answer 2: Child abuse is a common differential in cases of OI as both present with suspicious and atypical bone fractures. However, this patient also presents with signs of hearing loss and thus OI is a more likely explanation. Answer 4: Low levels of phosphate, or hypophosphatasia, may also cause brittle bones. It is a rare autosomal disease that results in abnormal mineralization of bone and dental tissues. However, patients with this disease would have decreased serum concentrations of alkaline phosphatase. Answer 5: Low levels of vitamin D, or rickets, can cause bone deformities and defective bone mineralization. However, hearing loss typically does not occur. It is also worth noting that most food products in the United States are fortified with vitamin D and thus this condition is rare. Bullet Summary: Osteogenesis imperfecta is due to mutations of genes that codes for type 1 collagen and presents with atypical fractures, blue sclerae, hearing loss, and easy bruising.

An 11-year-old girl presents to her primary care physician because she has been having difficulty hearing her teachers at school. She says that the difficulty hearing started about a year ago, and it has slowly been getting worse. Her past medical history is significant for multiple fractures in both her upper and lower extremities. She also recently had a growth spurt and says that her friends say she is tall and lanky. A mutation in a gene that produces which of the following protein is most likely associated with this patient's condition? A. Fibrillin B. Fibroblast growth factor receptor C. Type 1 collagen D. Type 3 collagen E. Type 4 collagen

C, Osteogenesis Imperfecta / Brittle Bone Disease This patient with hearing loss and recurrent fractures most likely has osteogenesis imperfecta, which is caused by a mutation in the genes that encode type 1 collagen. Osteogenesis imperfecta is caused by a defect in the production of type I collagen leading to abnormal, fragile bones that fracture easily. It is colloquially known as "brittle bone disease" and can often be mistaken for child abuse in young children. The pathogenesis by which mutations in type 1 collagen lead to recurrent fractures is that this collagen subtype is required for bone and tendon formation. Hearing loss can also result because type 1 collagen is required for the conductive pathways in the middle ear. Therefore damage to the ossicles or their sensorineural connections can lead to hearing loss. Incorrect Answers: Answer 1: Fibrillin is mutated in Marfan syndrome, which can present with a lanky physique and ocular/hearing problems; however, it wouldn't explain the recurrent fractures seen in this patient. Answer 2: Fibroblast growth factor receptor is mutated in achondroplasia; however, this disorder would present with short stature. Answer 4: Type 3 collagen is mutated in certain forms of Ehler-Danlos syndrome, which can present with joint dislocation and hearing loss; however, this disorder would not explain the recurrent fractures in this patient. Answer 5: Type 4 collagen is mutated in Alport syndrome, which would present with kidney and eye problems rather than fractures. Bullet Summary: Osteogenesis imperfecta presents with recurrent fractures and hearing loss.

A 5-year-old girl accompanied by her mother presents to the emergency department after suffering a fall on the elementary school playground. Her mother reports that a child on the playground pushed her daughter who fell on her right side, after which she screamed and was found clutching her right leg. The girl's past medical history is significant for a fracture of the left femur and right radius over the past 2 years and an auditory deficit requiring hearing aid use starting 6 months ago. Inspection reveals a relatively short girl in moderate distress. She has brown opalescent teeth. She refuses to bear weight on her right lower extremity. Radiography of the right lower extremity reveals a femoral midshaft fracture. Which of the following is the most likely etiology of the patient's condition? A. Decreased cystathionine beta synthase activity B. Defective type I collagen production C. Fibrillin gene defect D. Type III collagen gene defect E. Vitamin D deficiency

C, Osteogenesis Imperfecta / Brittle Bone Disease This patient's history of multiple fractures, hearing loss, and brown opalescent teeth is most consistent with a diagnosis of osteogenesis imperfecta (OI) which is due to defective type I collagen production. OI is a rare disorder due to defective production of type I collagen. Type I collagen is the most abundant type of collagen in the body, and is expressed in areas where high tensile strength is required such as bones, tendons, fascia, teeth (dentin), and skin. As a result, patients with OI suffer frequent fractures and may demonstrate other findings such as bowing of the distal extremities, blue sclera, brown opalescent teeth, and early onset hearing loss. Bisphosphonate therapy may be employed to increase bone density and help prevent fractures in these patients. Incorrect Answers: Answer 1: Decreased cystathionine beta synthase activity is seen in patients with homocystinuria. These patients may present with Marfanoid habitus, downward lens dislocations, intellectual disability, and recurrent thromboses. Frequent fractures, hearing loss, and abnormal dentition are not features of homocystinuria. Answer 3: Fibrillin gene mutations are seen in Marfan syndrome. These patients will present with a Marfanoid habitus, upward lens dislocations, and ascending aortic aneurysms. Frequent fractures, hearing loss, and abnormal dentition are not features of Marfan syndrome. Answer 4: Type III collagen gene defects are seen in Ehlers-Danlos type IV (vascular type). These patients will have a phenotype that demonstrates thin translucent skin along with a high risk of arterial aneurysm formation and rupture. Frequent fractures, hearing loss, and abnormal dentition are not features of vascular Ehlers-Danlos. Answer 5: Vitamin D deficiency in children can lead to rickets (osteomalacia). Like OI, these patients also have an increased risk of fracture and may demonstrate bowing of the distal extremities on physical examination. However, the presence of early onset hearing loss and brown, opalescent teeth make OI a more likely diagnosis in this patient. Bullet Summary: Osteogenesis imperfecta is due to a defect in type I collagen production which will result in frequent fractures, bowing of the distal extremities, blue sclera, early onset hearing loss, and brown opalescent teeth.

A 19-year-old African American man presents to his primary care provider for left leg pain. He reports a 2-month history of slowly progressive pain in the middle of his left leg. The pain is dull and gets much worse at night. He took some over-the-counter aspirin which seemed to improve his pain temporarily. His past medical history is notable for rheumatoid arthritis and asthma. He takes methotrexate and uses an albuterol inhaler as needed. His family history is notable for gout and rheumatoid arthritis in his father. On exam, he has mild localized tenderness to palpation along the anteromedial aspect of the tibial shaft. He has 5/5 strength throughout his left lower extremity with 2+ patellar and Achilles reflexes. Sensation to light touch is intact in the saphenous, sural, deep peroneal, superficial peroneal, and tibial distributions. A radiograph of the patient's right leg is shown in Figure A. This patient most likely has which of the following lesions? A. Osteoblastoma B. Osteoclastoma C. Osteoid osteoma D. Osteochondroma E. Enchondroma

C, Osteoid Osteoma The patient in this vignette presents with a painful lesion in the proximal tibia with pain that is worse in the evening and resolves with a non-steroidal anti-inflammatory drug (NSAID). This presentation is suggestive of osteoid osteoma. Osteoid osteomas are benign bone lesions that typically arise from the proximal femur, tibial diaphysis, and vertebrae. They are more common in males and normally present in children and young adults. Notably, the lesions are most painful at night and the pain is relieved by NSAIDs. Figure A demonstrates the characteristic appearance of an osteoid osteoma on a lateral radiograph of the tibia and fibula. Note a small reactive lesion with a radiolucent core in the posterior tibial diaphysis. This appearance is due to a tumor nidus of woven bone and osteoid with osteoblastic rimming that is surrounded by reactive bone and vascularity. Incorrect Answers: Answer 1: Osteoblastomas are morphologically similar to osteoid osteomas but are generally larger (> 2 cm while osteoid osteomas are < 2 cm). Osteoblastomas most commonly arise in the vertebral column. However, unlike osteoid ostomas, the pain in osteoblastomas generally does not improve with NSAIDs. Answer 2: Osteoclastomas (giant cell tumors) are locally aggressive but benign lesions that arise from osteoclasts. They appear to have a "soap bubble" appearance representative of a cavity lesions. This is due to increased osteoclastic activity and therefore increased bone breakdown. Due to their size and rapid growth, osteoclastomas are often painful. Answer 4: Osteochondromas (exostoses) are the most common type of benign bone tumor. Osteochondromas contain both bone and cartilage cap and most frequently arise as lateral projects from the growth plate. Most ostechondromas are asymptomatic unless they cause mechanical or compressive symptoms when they are very large. Answer 5: Enchondromas are purely cartilaginous neoplasms found in intramedullary bone. Enchondromas are most commonly found in the hands and feet. Enchondromas are often asymptomatic and found incidentally on radiographs. Bullet Summary: Osteoid osteomas are benign and painful bone lesions that typically present with progressively worsening pain that improves with NSAIDs.

A 65-year-old woman arrives for her annual physical. She has no specific complaints. She has seasonal allergies and takes loratadine. She had a cholecystectomy 15 years ago. Her last menstrual period was 9 years ago. Both her mother and her maternal aunt had breast cancer. A physical examination is unremarkable. The patient is given the pneumococcal conjugate vaccine and the shingles vaccine. A dual-energy x-ray absorptiometry (DEXA) scan is obtained. Her T-score is -2.6. She is prescribed a new medication. The next month the patient returns to her primary care physician complaining of hot flashes. Which of the following is the most likely medication the patient was prescribed? A. Alendronate B. Denosumab C. Raloxifene D. Teriparatide E. Zoledronic acid

C, Osteoporosis The patient is presenting with hot flashes after recently starting a medication for postmenopausal osteoporosis. Hot flashes are a side effect associated with raloxifene. Raloxifene is a selective estrogen-receptor modifier. It is an agonist to estrogen receptors in bone and prevents bone resorption. It is also an antagonist to estrogen receptors in the uterus and breasts and may prevent breast and uterine cancer. Common adverse effects of raloxifene include hot flashes and venous thromboembolism. Tamoxifen is also a selective estrogen-receptor modulator, but unlike raloxifene, it has an increased risk of endometrial carcinoma as it is a partial agonist in the endometrium. Incorrect Answers: Answer 1: Alendronate is a bisphosphonate. Bisphosphonates work by binding to osteoclasts and hydroxyapatite causing osteoclast apoptosis and prevention of further osteoclast recruitment. Adverse effects of bisphosphonates include bone pain (i.e., ostealgia), osteonecrosis, esophagitis, other gastrointestinal complaints, and flu-like symptoms. Answer 2: Denosumab is a monoclonal antibody that binds to RANK-L. It prevents the activation of osteoclasts, thereby reducing bone resorption. Adverse effects include arthralgias, myalgias, an increased risk of infection, and osteonecrosis. Answer 4: Teriparatide is a parathyroid hormone analog. It is often used for severe osteoporosis (T < -3.5). Adverse effects are limb pain, headache, hypotension, nausea, and a theoretical risk of osteosarcoma. Answer 5: Zoledronic acid is a bisphosphonate. Adverse effects of bisphosphonates include bone pain (i.e., ostealgia), osteonecrosis, esophagitis, other gastrointestinal complaints, and flu-like symptoms. Bullet Summary: Adverse effects of raloxifene include hot flashes and venous thromboembolism.

A 77-year-old female presents to the emergency room complaining of right hip pain and inability to move her hip. She is accompanied by her daughter who reports that the patient fell while walking up the stairs at home. The patient has a history of a wrist fracture and vertebral compression fracture within the past three years. Prior to her fall, the patient was feeling well and denied any focal bone pain, muscle pain, weakness, or tingling. On exam, her right leg is shortened, abducted, and externally rotated. A radiograph of her right hip is shown in Figure A. Which of the following sets of lab findings is most likely to be found in this patient? A. Decreased calcium, decreased phosphate, elevated alkaline phosphatase, elevated parathyroid hormone B. Normal calcium, normal phosphate, elevated alkaline phosphatase, normal parathyroid hormone C. Normal calcium, normal phosphate, normal alkaline phosphatase, normal parathyroid hormone D. Elevated calcium, decreased phosphate, elevated alkaline phosphatase, elevated parathyroid hormone E. Decreased calcium, increased phosphate, normal alkaline phosphatase, decreased parathyroid hormone

C, Osteoporosis The most likely diagnosis for this patient is hip fracture in the setting of underlying osteoporosis. Serum findings are normal (calcium, phosphate, alkaline phosphatase, and parathyroid hormone) in osteoporosis. Osteoporosis is a common condition leading to diffuse bone demineralization. Elderly patients, especially women, are at highest risk of osteoporosis. Since osteoporosis weakens bone integrity, it often presents acutely in the form of hip fracture, vertebral compression fracture, or distal radius fracture. Given this patient's age, female sex, and history of vertebral compression fracture/wrist fracture, she likely has osteoporosis that predisposed her to hip fracture. Hip fractures often present with severe hip pain and an inability to move the hip. Depending on the location of the fracture, the affected leg is often shortened, abducted, and externally rotated. Importantly, serum findings are normal in osteoporosis. Figure A shows an anterior-posterior radiograph of the right hip with a fracture line along the femoral neck. Bone demineralization can be noted throughout the femoral neck. Incorrect Answers: Answer 1: Decreased calcium and decreased phosphate can be found in rickets and osteomalacia, which are conditions associated with vitamin D deficiency due to either dietary deficiency or impaired metabolism. These conditions typically present with bone demineralization and increased risk of fracture. This patient has no history of dietary deficiency or metabolic abnormalities and her history of vertebral compression and wrist fractures is more consistent with osteoporosis. Answer 2: An isolated elevated alkaline phosphatase with otherwise normal serum findings is characteristic of Paget disease of the bone. Paget disease of the bone is a disorder characterized by localized bone remodeling, including increased osteoclastic reabsorption and osteoblastic bone formation. Bone pain is common. Given that this patient has no prior history of localized bone pain and has multiple factors in her history consistent with osteoporosis, she likely does not have Paget's disease. Answer 4: Elevated calcium, decreased phosphate, elevated alkaline phosphatase, and elevated parathyroid hormone are characteristic findings of primary hyperparathyroidism. Parathyroid hormone acts to increase bone resorption, stimulate calcium reabsorption in the kidney, and inhibit phosphate reabsorption in the kidney. The net effect of excess parathyroid hormone is to increase levels of calcium and alkaline phosphate while decreasing phosphate levels. Primary hyperparathyroidism is often asymptomatic. Symptoms are generally caused by hypercalcemia and include weakness, fatigue, bone pain, and muscle pain. Answer 5: Decreased calcium, increased phosphate, normal alkaline phosphatase, and decreased parathyroid hormone are characteristic findings of hypoparathyroidism. Due to decreased levels of parathyroid hormone, bone resorption is reduced and the kidney excretes less phosphate. Symptoms are generally caused by hypocalcemia and include tetanus, cramping, and cardiac arrhythmias. Bullet Summary: Osteoporosis is a condition associated with diffuse bone demineralization that presents with hip fracture, vertebral compression fracture, and distal radius fracture in the elderly. Serum findings are normal in osteoporosis.

A 64-year-old female presents to her primary care physician for an annual checkup. She states that her health is adequate, but she has not been doing well since her husband died last year. She is able to get by but admits to having trouble caring for herself, cooking, cleaning, and paying her bills. The patient complains of diffuse muscle aches and pains. She has a past medical history of anxiety and seasonal affective disorder. She is not currently taking any medications. On physical exam, you note a gaunt woman with a depressed affect. You note that her body mass index (BMI) and height have both decreased since her last annual visit. On physical exam, her cardiac, pulmonary, and abdominal exams are within normal limits. Lab work is drawn and is as follows: Serum: Na+: 135 mEq/L K+: 3.7 mEq/L Cl-: 100 mEq/L HCO3-: 23 mEq/L Urea nitrogen: 7 mg/dL Glucose: 70 mg/dL Creatinine: 0.8 mg/dL Ca2+: 8.0 mg/dL Mg2+: 1.5 mEq/L Parathyroid hormone: 855 pg/mL Alkaline phosphatase: 135 U/L Phosphorus: 2.6 mg/dL Hemoglobin: 14 g/dL Hematocrit: 36% Platelet count: 187,000/mm^3 Leukocyte count: 4,700/mm^3 What is the most likely diagnosis? A. Osteopenia B. Osteoporosis C. Osteomalacia D. Hyperparathyroidism E. Clinical malnutrition

C, Osteoporosis This patient is presenting with a change in her height (reflecting minor fractures), diffuse muscle aches, and abnormalities in her laboratory values suggesting a diagnosis of osteomalacia. Osteomalacia is a qualitative defect in bone mineralization that typically occurs secondary to malnutrition (though it can also occur secondary to chronic anticonvulsant use, renal failure, or from malabsorption syndromes). These patients are fundamentally deficient in vitamin D and will have low levels of 1,25-OH vitamin D. This deficiency leads to decreased phosphate reabsorption (hypophosphatemia), decreased calcium absorption (hypocalcemia), and increased parathyroid hormone (PTH) and alkaline phosphatase levels secondary to the low calcium levels. Treatment involves replacement of vitamin D and calcium if poor nutrition is the sole cause. Incorrect Answers: Answer 1 & 2: Osteopenia (mild) and osteoporosis (severe) are on the same spectrum of disease and are a quantitative defect in bone mineralization. Though this patient could have osteopenia/osteoporosis given her decreasing height (compression fractures), her laboratory values suggest a diagnosis of osteomalacia. In osteopenia and osteoporosis, calcium, phosphorus, PTH, and alkaline phosphatase levels are relatively normal as compared to this patient's lab values. Answer 4: Hyperparathyroidism typically presents with hypercalcemia, hypophosphatemia, increased PTH levels, and decreased bone mineralization. This patient's laboratory values to not reflect these changes. Answer 5: Clinical malnutrition would present with severe muscle wasting and weight loss. While it is possible that this patient is malnourished, her laboratory values support a diagnosis of osteomalacia. Of note, though the question states that the patient's BMI has decreased, not enough information is given to determine if she is even underweight. Bullet Summary: Osteomalacia (qualitative defect) presents with pseudofractures, muscle pain, and lab abnormalities (hypocalcemia, hypophosphatemia, increased PTH, increased alkaline phosphatase levels) in contrast to osteoporosis (quantitative defect without lab abnormalities).

A 69-year-old woman comes to the emergency department for pain in her right wrist following a fall. She was walking out from the grocery store when she tripped on something and fell forward, landing on her right hand as she tried to brace her fall. She denies bleeding, paresthesia, or loss of sensation, but endorses a 7/10, sharp, stabbing pain in her right wrist. Her past medical history is significant for obesity, hypertension controlled with lisinopril, and myositis managed with glucocorticoids. She used to smoke 1-2 cigarettes over the weekend when she was in her thirties. The patient reports that she weight lifts 3 times a week. A physical examination demonstrates a swollen wrist and tenderness upon palpation along the right wrist and distal forearm. A radiograph of the right wrist is obtained and is shown in Figure A. Which of the following is a risk factor for her condition? A. Exercise B. Hypertension C. Medication use D. Obesity E. Smoking history

C, Osteoporosis This patient is suffering from a Colles fracture likely secondary to osteoporosis (e.g., pathological bone fracture following a minor fall). Use of glucocorticoids is a major risk factor for osteoporosis. Osteoporosis is characterized by low bone mass, microarchitectural disruption, and skeletal fragility that culminates in an overall decrease in bone strength. Patients with osteoporosis are thus at an increased risk of fracture. A clinical diagnosis of osteoporosis is often made during an annual check-up or following a fracture as there are no clinical manifestations until a fracture. The gold standard of diagnosis involves dual-energy x-ray absorptiometry (DEXA), where a T-score of < -2.5 is consistent with osteoporosis. Some common risk factors include advancing age, previous fractures, glucocorticoid use, parental history of hip fracture, low body weight, current cigarette smoking, alcohol abuse, and rheumatoid arthritis. Of note, steroids inhibit bone formation and can thus contribute to the development osteoporosis. Figure/Illustration A is a radiograph of the right wrist demonstrating a Colles fracture (arrow), an extraarticular fracture of the distal radius with dorsal displacement of distal fracture fragment. Incorrect Answers: Answer 1: Exercise, especially weight-bearing exercise, is protective of osteoporosis. Answer 2: Hypertension is not a key risk factor for osteoporosis. Answer 4: Obesity is not a key risk factor for osteoporosis Answer 5: Smoking history is a risk factor for osteoporosis if the patient is a current smoker. In this patient reporting a remote history of mild pack years, this is non-contributory. Bullet Summary: Important risk factors for osteoporosis include advancing age, previous fractures, glucocorticoid use, parental history of hip fracture, low body weight, current cigarette smoking, alcohol abuse, and rheumatoid arthritis.

A 14-year-old previously healthy male presents to your office with complaints of two months of dull, aching pain in his right leg that has recently become worse. While he experiences the pain constantly, he notes that it is sometimes so bad at night that it awakes him from his sleep. Though an avid soccer player, he has been unable to participate this season due to the pain and difficulty running. He cannot think of any particular trauma that might have precipitated this pain. Vitals are all within normal range. Physical examination reveals localized tenderness below the knee with mild swelling and reduced range of motion. You order an X-ray of the painful limb, which is shown in Figure A. What is the most likely diagnosis? A. Osteomyelitis B. Ewing sarcoma C. Osteosarcoma D. Growing pains E. Slipped capital femoral epiphysis

C, Osteosarcoma This patient's history, presentation, and radiographic findings are most consistent with a diagnosis of osteosarcoma. Osteosarcoma is a primary malignant tumor of bone characterized by its osteoblastic differentiation and its production of tumor osteoid. Though relatively uncommon, osteosarcoma is the most common primary bone malignancy in the pediatric and young adult patient populations. Though they can emerge anywhere, the most common site of origin of osteosarcomas in children is in the metaphysis of long bones with the two most common sites being the distal femur and proximal tibia. Wittig et al. discuss the necessary multidisciplinary approach between primary care physicians, orthopedic specialists, oncologists, radiologists, and pathologists in navigating the diagnosis and management of osteosarcoma. They describe how the classic presentation in children is a dull, aching pain, often severe at night, thereby often mimicking growing pains. They describe how physical exam can reveal such findings as local tenderness, swelling, deformity, and restricted range of motion. They further discuss the common radiographic finding of mixed sclerotic and lytic lesion, most commonly located in the metaphysis in the bone of origin. Daft et al. sought to examine how VEGF plays a role in providing the high vascular density required by osteosarcoma for its growth and proliferation. They showed how alpha-Ca2+/calmodulin kinase II (alpha-CaMKII) serves as a positive regulator of VEGF, and conclude that dual therapy to inhibit alpha-CaMKII and VEGF is a potential future avenue for innovation of therapy against osteosarcoma. Figure A illustrates the classic radiographic findings of sunburst pattern due to the mixed sclerotic and lytic lesion and the pathognomonic "Codman's triangle" that results from periosteal reaction. Illustration A depicts the tumor osteoid characteristic of all subtypes of osteosarcoma. Incorrect Answers: Answer 1: Osteomyelitis, while frequently occurring in the long bones and associated with localized pain and swelling, generally presents with an associated fever in the pediatric population. Moreover, this patient's more indolent course and lack of inciting event are more consistent with a diagnosis of osteosarcoma. Answer 2: Ewing's sarcoma more often arises in the diaphysis of long bones as compared to osteosarcoma, which more often arises in the metaphysis of long bones. Additionally, whereas osteosarcoma is associated with a "sunburst" appearance and/or Codman's triangle on X-ray, Ewing's sarcoma is associated with an radiographic "onion-skinning" appearance. Answer 4: Growing pains are a common condition of unknown etiology in the pediatric population. Pain is generally bilateral and characteristically present in evening hours and gone by the morning with no associated evidence of inflammation or other lesion, with the condition self-resolving in late childhood. Answer 5: Slipped capital femoral epiphysis, most commonly associated with childhood obesity, would present with reduced range and motion and gradual development of pain, though X-ray would reveal a fracture through the proximal femoral epiphysis.

69-year-old man with no significant past medical history except for hypertension receives an X-ray of his skull (Figure A) after his car was rear-ended in a parking lot resulting in neck soreness. No fracture but what is the likely diagnosis of this incidental finding? A. Bony metastases B. Osteoarthritis C. Paget's disease (osteitis deformans) D. Osteitis fibrosa cystica E. Osteomalacia

C, Paget Disease of Bone (Osteitis Deformans) Paget's disease (osteitis deformans) is a metabolic bone disease that causes sclerotic and deformed bones that may demonstrate "cotton wool spots" and long bone bowing on x-ray. Paget disease is the most common bone disorder after osteoporosis, with increased incidence in >50 years old. Paget's disease is caused by accelerated bone resorption followed by deposition of dense, chaotic bone matrix mineralization resulting sclerosis and deformation. Up to 70% of patients are asymptomatic and others develop complications such as bone pain, deformity, nerve compression syndromes and fragility fractures. Elevated alkaline phosphatase is used as a biomarker of the disease. Patients may present with metabolic abnormalities such as hypercalcemia, hypercalciuria and hyperuricemia. Josse et al. discuss the types of common radiologic findings in patients with Paget's disease including: cotton wool spots, flame-shaped lesions in long bones, osteolytic lesions near thickened lesions, sclerotic bone, bowed limbs, transverse fractures. Among some of the most commonly affected limbs are the pelvis (72%), lumbar spine (58%), femur (55%), thoracic spine (45%) and skull (42%). Delmas et al. discuss indications for therapy. Those with bone pain or those at risk of complications, (e.g: hearing loss or spinal cord compression) should receive anti-resorptive therapies. Second-generation oral bisphosphonate, such as alendronate, are a first line therapy. IV pamidronate, etidronate are acceptable alternatives. Calcitonin may be used to decrease bone pain rapidly. Figure A demonstrates the "cotton wool spots", ill-defined sclerotic and lucent areas throughout that is a finding of Paget disease . The cortex is thickened and irregular. Incorrect answers: Answer 1: Bony metastases are most commonly associated with primary tumors from breast cancer, prostate cancer and lung cancer. Common sites of bone metastasis include spine, pelvis, ribs, skull and proximal femur. This Patient has no symptoms suspicious for primary cancer. Answer 2: Osteoarthritis is a common degenerative joint disease that results from breakdown of joint cartilage and underlying bone. Commonly affected joints include fingers, low back, knees and hips. Answer 4: Osteitis fibrosa cystica is a skeletal disorder caused by hyperparathyroidism. Excess parathyroid hormone increases osteoclastic bone resorption. Patients have elevated serum calcium levels and form cyst-like brown tumors in and around bone. Answer 5: Osteomalacia is the softening of bones caused by defective bone mineralization, most commonly caused by vitamin D deficiency. In adults, symmetric pain usually begins in the lumbar spine.

A 63-year-old male presents to his family physician with a chief complaint of right knee and hip pain as well as pain in his left upper arm. He also reports frequent headaches and notes that he has had to adjust the headband on his favorite baseball cap to a larger size after it began feeling tight earlier this year. His wife, who accompanied the patient to the visit, complains that his hearing has deteriorated noticeably over the past several months. Physical examination shows bilateral knee effusions as well as tenderness to palpation of the left shoulder as well as the right hip and lower leg. Serologic testing reveals an elevated alkaline phosphate level but normal vitamin D, calcium, and PTH. A bone scan is conducted which is demonstrated in Figure A. Which of the following is the first-line treatment of this patient's condition? A. Teriparatide B. Estrogen C. Zoledronate D. Dexamethasone E. Methotrexate

C, Paget Disease of Bone (Osteitis Deformans) This patient is suffering from symptomatic Pagets bone disease. The first-line treatment for this condition is an aminobisphosphonate (Zoledronate). Pagets bone disease is an autosomal dominant disorder of abnormal bone remodeling that involves increased osteoblastic and osteoclastic activity. Bisphosponates act to inhibit osteoclasts and therefore slow resorption of bone. Diagnosis is often achieved through an incidental finding on a radiograph (shows thickened bone, bowing, and cotton wool spots on the skull). However, bone scan is the most sensitive diagnostic test. Less than one percent of patients will experience progression to Pagets sarcoma. Schneider et al. discuss the diagnosis and management of Pagets bone disease. The condition is frequently asymptomatic; however, when symptomatic, the presentation of the disease is dependent on the specific bones involved. Bone/joint pain as well as hearing, vision, cardiac (high output congestive heart failure), and oncologic (progression to secondary osteosarcoma) issues may arise in Pagets. Wat reviews the use of bisphosphonates in the management of Pagets bone disease. The use of bisphosphonates should be limited to symptomatic patients to ensure that the benefits outweigh the risks associated with this class of medications. Complications associated with bisphosphonate therapy can include atypical femoral fracture and/or osteonecrosis of the jaw. Calcium and vitamin D supplementation should precede and accompany bisphosphonate therapy to prevent the development of hypocalcemia. Figure A demonstrates a typical bone scan seen in Pagets disease with increased uptake. Illustration A shows the radiographic appearance of bone that may be seen in Pagets disease. Illustration B summarizes some of the sequela and complications associated with Pagets disease. Incorrect Answers: Answer 1: Teriparatide, recombinant parathyroid hormone, is used in the treatment of osteoporosis; it activates osteoblasts to stimulate bone formation. Answer 2: Estrogen hormone therapy may be used for the treatment of osteoporosis and other symptoms in menopausal women; it should not be used in pre-menopausal women or men. Answer 4: Glucocorticoid therapy does not have a role in the treatment of Pagets disease. Answer 5: Methotrexate is a third-line therapy for the treatment of Pagets disease; it should only be tried after bisphosphonates and calcitonin have been attempted.

A 67-year-old man comes to the clinic complaining of left shin pain for the past 2 months. He reports that ever since he injured that leg from a fall he has been experiencing this 8/10, stabbing pain at his left upper shin that comes and goes. It has been progressively worsening to the point that it is affecting his daily life (e.g., walking to the grocery store). The patient denies fever, chills, weight loss, chest pain, shortness of breath, weakness, or loss of sensation. A physical examination demonstrates what appears to be a soft tissue mass at the proximal tibia. A radiograph of the left leg is shown in Figure A. Which of the following is a significant risk factor for this patient's disease? A. Growing pains during childhood B. Osteochondroma C. Paget disease of the bone D. Rheumatoid arthritis E. Sporadic retinoblastoma

C, osteosarcoma This patient has osteosarcoma (e.g., persistent pain, soft tissue mass, and sunburst pattern on radiograph). Key risk factors for the development of osteosarcoma includes Paget disease of bone, bone infarcts, prior radiation, familial retinoblastoma, and Li-Fraumeni syndrome. Osteosarcoma is one of the most common malignant bone tumors and is characterized by the production of osteoid (immature bone) by malignant cells. The disease has a bimodal age distribution and peaks in early adolescence and in adults over the age of 65. it is worth noting that in the United States, over half of osteosarcomas over the age of 60 are secondary to sarcomatous transformation of Paget disease of bone. Most patients present with localized pain that lasts several months most commonly involving the distal femur, proximal tibia, proximal humerus, and middle/proximal femur. Systemic symptoms such as fever, weight loss, and malaise are usually absent. Treatment involves aggressive surgical resection and chemotherapy. Figure/Illustration A is a clinical radiograph of the left leg demonstrating the sunburst sign (arrow) characteristic of osteosarcoma at the proximal tibia. Incorrect Answers: Answer 1: Growing pains during childhood is a benign physiological symptom that some adolescents may experience during growth spurts. This does not increase one's risk of developing osteosarcoma. Answer 2: Osteochondroma is the most common benign bone tumor and can transform to chondrosarcoma on rare occasions. Answer 4: Rheumatoid arthritis is an autoimmune condition that presents with polyarthritis and joint changes. It does not increase one's risk of developing osteosarcoma. Answer 5: Sporadic retinoblastoma is due to sporadic mutation of the RB2 gene in non-germline cells. Since the mutation does not affect germline cells, the association between familial retinoblastoma and osteosarcoma does not apply. Bullet Summary: Paget disease of the bone is a significant risk factor for osteosarcoma in patients over the age of 60.

A 13-year-old Caucasian boy presents with his mother to the pediatrician's office for right arm pain. He reports that he has noticed slowly progressive pain and swelling over the middle of his right arm over the past two months. He denies any recent trauma to the area. He is otherwise healthy and takes no medications. His temperature is 99.8°F (37.7°C), blood pressure is 115/65 mmHg, pulse is 89/min, and respirations are 16/min. On exam, there is swelling and tenderness overlying the right humeral diaphysis. Laboratory evaluation is shown below: Serum: Hemoglobin: 15.1 g/dL Hematocrit: 42% Leukocyte count: 13,000/mm^3 with normal differential Platelet count: 210,000/mm^3 Erythrocyte sedimentation rate: 40 mm/hr (normal 0-22 mm/hr) A radiograph of the patient's right arm is shown in Figure A. Which of the following genetic mutations is associated with this patient's condition? A. t(8;14) B. t(9;22) C. t(11;14) D. t(11;22) E. t(14;18)

D, Ewing sarcoma The patient in this vignette presents with an irregular humeral diaphyseal tumor with leukocytosis and elevated inflammatory markers suggestive of Ewing sarcoma. Ewing sarcoma is associated with a t(11;22) translocation. Ewing sarcoma is a malignant bone tumor that typically affects children, especially males, less than 15 years old. Ewing sarcoma typically arises in the diaphysis (midshaft) of long bones (e.g., femur, tibia, and humerus) and may present with localized pain, fever, leukocytosis, and elevated inflammatory markers. On radiographic imaging, a large destructive lesion will be seen in the diaphysis of a long bone. Concentric layers of new bone growth can lead to an "onion skinning" or "sunburst" appearance on radiograph. Ewing sarcoma is caused by a t(11;22) translocation which creates a fusion protein between RNA-binding protein EWS and Friend leukemia integration 1 transcription factor (FLI1). Figure A demonstrates periosteal reaction in the humeral diaphysis leading to an "onion skinning" appearance consistent with Ewing sarcoma. Incorrect Answers: Answer 1: t(8;14) is associated with Burkitt lymphoma, a form of non-Hodgkin lymphoma seen in the pediatric population. This translocation forms a fusion gene between c-myc (chromosome 8) and the Ig heavy chain (chromosome 14). Answer 2: t(9;22) is associated with chronic myeloid leukemia, a form of leukemia seen primarily in the elderly. This translocation forms the "Philadelphia chromosome" which contains a fusion gene between ABL (chromosome 9) and BCR (chromosome 22). Answer 3: t(11;14) is associated with mantle cell lymphoma, a form of non-Hodgkin lymphoma seen primarily in the elderly. This translocation forms a fusion gene between cyclin D (chromosome 11) and the Ig heavy chain (chromosome 14). Answer 5: t(14;18) is associated with follicular lymphoma, a form of non-Hodgkin lymphoma seen primarily in the elderly. This translocation forms a fusion gene between the Ig heavy chain (chromosome 14) and bcl-2 (chromosome 18). Bullet Summary: Ewing sarcoma is a malignant bone tumor that typically affects children less than 15 years old, arises in long bone diaphyses, and is associated with a t(11;22) translocation

A 72-year-old man presents to his primary care physician for his annual exam. He has a very stoic personality and says that he is generally very healthy and has "the normal aches and pains of old age." On further probing, you learn that he does have pretty significant back and hip pain that worsens throughout the day. On physical exam you note bony enlargement of the distal interphalangeal joints bilaterally. Which of the following is the likely cause of his symptoms? A. Gout B. Pseudogout C. Rheumatoid arthritis D. Osteoarthritis E. Osteopaenia

D, Osteoarthritis This man has classic findings of osteoarthritis, which is characterized by DIP joint osteophyte/nodule formation. Osteoarthritis is a disease of chronic wear that ultimately results in the destruction of articular cartilage and the formation of subchondral bone and osteophytes. Weight-bearing joints, such as the back and hips are most commonly affected, as in this case. Unlike rheumatoid arthritis, the symptoms of osteoarthritis worsen throughout the day and are more commonly seen at the DIP joints. Sinusas describes the risk factors, diagnosis, and treatment of osteoarthritis. Genetics, female sex, trauma, advancing age, and obesity predispose patients to osteoarthritis, the diagnosis of which is based largely on a history of joint pain worsened by movement that affects activities of daily living. Pharmacologic treatment should begin with acetaminophen and progress to nonsteroidal anti-inflammatory drugs if necessary. It might seem counter-intuitive, but exercise/physical therapy is a useful adjunct to treatment and has been shown to reduce pain and disability. McHughes and Lipman describe alternative therapies to NSAIDs, opioids, and surgery for the management of osteoarthritis, which include tramadol and intra-articular injections of steroids and hyaluronic acid. There is evidence to support the familiar over-the-counter nutraceutical glucosamine as an adjunct in treating osteoarthritis; however, evidence is lacking to support the use of chondroitin, S-adenosyl-methionine, or dimethyl sulfoxide in osteoarthritis pain management. Some patients choose to pursue these drugs despite the absence of evidence with mixed reviews. Figure A shows the classic bony enlargements of the DIP joints, known as Heberden's nodes. Illustration A shows an x-ray of a hand with the classic findings of osteoarthritis including osteophyte formation, subchondral bone deposition and joint space narrowing. Incorrect Responses: Answer 1: Gout results in the painful swelling of a joint, most commonly the metatarsophalangeal joint that results from precipitation of monosodium urate crystals. Answer 2: Pseudogout is similar to gout, but results from precipitation of calcium pyrophosphate crystals. Answer 3: Rheumatoid arthritis is an autoimmune inflammatory disorder that most commonly affects the PIP joints. The symptoms of rheumatoid arthritis often improve throughout the day. Answer 5: Osteopenia is the loss of bone density secondary to hormone-related calcium resorption that is especially common in postmenopausal women. It is otherwise asymptomatic but can predispose patients to fractures.

A 67-year-old Caucasian female presents to her primary care physician after a screening DEXA scan reveals a T-score of -3.0. Laboratory work-up reveals normal serum calcium, phosphate, vitamin D, and PTH levels. She smokes 1-2 cigarettes per day. Which of the following measures would have reduced this patient's risk of developing osteoporosis? A. Reduced physical activity to decrease the chance of a fall B. Initiating a swimming exercise program three days per week C. Intranasal calcitonin therapy D. Calcium and vitamin D supplementation E. Weight loss

D, Osteoporosis This patient with a DEXA T-score of less than -2.5 has osteoporosis. Interventions that decrease the risk of osteoporosis include: smoking cessation, a weight-bearing exercise regimen, vitamin D and calcium supplementation, or hormone replacement therapy. Primary osteoporosis (post-menopausal or senile types) is a reduction in bone mass and bone density due to increased osteoclastic bone resorption and decreased bone formation by osteoblasts in the absence of another disease process. It is most commonly seen in thin, postmenopausal, Caucasian or Asian women. Additional risk factors include smoking, caffeine and alcohol intake, family history, low calcium/vitamin D intake, and nulliparity. Jeremiah et al. discuss the screening for and workup of osteoporosis. The USPSTF recommends DEXA scans for all women age 65 and older as well as younger women determined to have an increased fracture risk. In new osteoporosis diagnoses, secondary causes should be ruled out by assessing laboratory values of vitamin D, calcium, creatinine, and TSH. Reid et al. review the benefits and risks of calcium supplementation. The authors argue that calcium supplementation has a negative risk-benefit ratio, with risks including gastrointestinal side effects, kidney stones, and increased myocardial infarction risk. In addition, many large meta-analyses have failed to demonstrate a significant reduction in fracture risk as a result of calcium supplementation. Illustration A shows a classification of osteoporosis based on DEXA scan T-score. Incorrect Answers: Answer 1: Although fall-precautions are important in preventing a fracture in any elderly person, weight-bearing exercises such as vigorous walking and/or weightlifting increase bone stress, which reduces the risk of osteoporosis. Answer 2: Swimming, although beneficial from a cardiovascular standpoint, reduces bone stress and does not prevent development and/or progression of osteoporosis. Answer 3: Intranasal calcitonin therapy is a last-line treatment option for osteoporosis; bisphosphonates and preventative measures should be attempted first. Answer 5: Weight loss, particularly greater than 10% of body weight, increases the risk of bone loss and osteoporosis.

A 72-year-old woman comes to you complaining of severe back pain. She reports that she slipped and fell in her shower at home. She also had a wrist fracture 2 years ago and an ankle fracture last year. The patient reports that she has been taking an over-the-counter vitamin D supplement. She has not had any diffuse bone pain. Figure A shows a radiograph of her spine. Labs values for parathyroid hormone, calcium, phosphate, and alkaline phosphate are normal. Which of the following is the most likely underlying abnormality? A. Subperiosteal bone resorption B. Vitamin D deficiency C. Excessive unmineralized osteoid D. Trabecular thinning E. Hypocalcemia

D, Osteoporosis This patient, with normal lab values, has osteoporosis, which is an age-related decrease in bone mass in which bony trabeculae become thinner. Osteoporosis is a common disorder of bone metabolism that can predispose to fracture. Osteoporosis is an age-related decrease in bone mass secondary to uncoupling of osteoclast-osteoblast activity. In osteoporosis, the existing bone is adequately mineralized. This is in contrast to osteomalacia, in which osteoid is inadequately mineralized. Findings in osteoporosis include thinned cortices and loss of trabecular bone. Figure A depicts a radiograph of an osteoporotic spine with two compression fractures labeled "1" and "3." Incorrect Answers: Answer 1: Subperiosteal bone resorption is associated with hyperparathyroidism. The patient would have an elevated parathyroid hormone in primary hyperparathyroidism which this patient does not. Answers 2-3: Vitamin D deficiency and excessive unmineralized osteoid are findings in osteomalacia. In osteomalacia, you would have decreased calcium and phosphate levels and increased levels of parathyroid hormone and alkaline phosphatase. This patients labs are all normal. Answer 5: Hypocalcemia is not seen in osteoporosis, which classically has normal serum calcium and vitamin D levels. Hypocalcemia is seen in osteomalacia. Bullet Summary: Osteoporosis is an age-related decrease in bone mass characterized by thinning of the bony trabeculae, and lab values for calcium, phosphate, alkaline phosphatase, and parathyroid hormone levels are normal.

A 67-year-old male presents to his primary care physician complaining of left hip pain for the past six months. He denies any trauma or recent falls. He is accompanied by his wife who reports that he has experienced progressive hearing loss over the same time period. The patient has also noticed that he is no longer able to fit into his favorite hat even though it previously fit well. A radiograph of the patient's pelvis is shown. Which of the following laboratory abnormalities is most likely to be found in this patient? A. Elevated serum parathyroid hormone B. Elevated serum calcium C. Decreased serum calcium D. Elevated serum alkaline phosphatase E. Decreased serum alkaline phosphatase

D, Paget Disease of Bone The most likely diagnosis for this patient is Paget's bone disease. This disease is associated with an isolated elevation in serum alkaline phosphatase with normal serum calcium, phosphate, and parathyroid hormone levels. Paget's bone disease typically presents in older adults with bone pain, decreased hearing, and increasing cranial diameter. It is caused by excessive bone resorption by osteoclasts followed by formation of disordered new bone by osteoblasts. This initially results in loss of bone mass that is eventually replaced by new bone that is structurally poor, which can be seen on imaging as expanded cortices and coarsened trabeculae. Serum findings in Paget's bone disease include an isolated elevated serum alkaline phosphatase with normal serum calcium, phosphate, and parathyroid hormone. Figure A shows a radiograph demonstrating an anterior-posterior view of the pelvis. The left femur is abnormal with expanded bony cortices and coarsened trabeculae representing disordered new bone formation. Incorrect Answers: Answer 1: Abnormally elevated serum parathyroid hormone can cause hyperparathyroidism, which leads to symptoms associated with hypercalcemia and hypophosphatemia. It does not typically present with bone pain, hearing loss, or increasing cranial diameter. Answer 2: Increased serum calcium (hypercalcemia) can be caused by several different conditions including hyperparathyroidism, acute renal failure, and granulomatous diseases. Signs and symptoms of hypercalcemia include fatigue, nausea, vomiting, constipation, muscle weakness, polyuria, and polydipsia. Answer 3: Decreased serum calcium (hypocalcemia) can be caused by several different conditions including vitamin D deficiency, hypoparathyroidism, hypoalbuminemia, and hypomagenesmia. Signs and symptoms of hypocalcemia include numbness, tingling, muscle cramps, hyperreflexia, and cardiac arrhythmias. Answer 5: Paget's bone disease is associated with an increased serum alkaline phosphatase due to increased bone turnover. Bullet Summary: Paget's bone disease is a condition characterized by increased osteoclastic bone resorption followed by abnormal bone formation by osteoblasts. It typically presents with bone pain, hearing loss, and increased cranial diameter and is associated with an isolated increase in serum alkaline phosphatase.

A 63-year-old man presents to his primary care provider for difficulty hearing. He reports developing significantly decreased hearing over the past 3-4 months that is worse in his right ear. He previously had good hearing and did not require a hearing aid. He is otherwise healthy and swims 1-2 miles per day. He has a distant history of multiple ear infections as a child. His temperature is 98.6°F (37°C), blood pressure is 140/85 mmHg, pulse is 82/min, and respirations are 19/min. Physical examination demonstrates a well-appearing male in no acute distress. He has no focal tenderness. Audiological examination demonstrates bilateral decreased hearing that is worse on the right. A radiograph of the patient's skull demonstrates multiple lytic and sclerotic lesions with cortical thickening and bony expansion. Serological analysis in this patient would likely reveal which of the following in Figure A? A. A B. B C. C D. D E. E

D, Paget Disease of Bone The patient in this vignette presents with hearing loss and mixed lytic-sclerotic bone lesions suggestive of Paget disease of bone. This condition is associated with an isolated increase in alkaline phosphatase with otherwise normal serological findings. Paget disease of bone is a progressive metabolic bone disorder characterized by abnormal bony remodeling. In this condition, excess bone resorption and formation results in the replacement of sturdy lamellar bone with immature woven bone. Most patients are asymptomatic but some of the more common symptoms include bone pain, joint pain, and hearing loss due to skull involvement leading to narrowing of the auditory foramen. Figure A is a table depicting the possible laboratory changes in this patient; see explanations in Incorrect Answers section for a further description. Incorrect Answers: Answer 1: Decreased calcium, decreased phosphate, increased alkaline phosphatase, and increased parathyroid hormone is consistent with serological findings seen in rickets or osteomalacia (A). Answer 2: Decreased calcium, increased phosphate, increased alkaline phosphatase, and increased parathyroid hormone is consistent with serological findings seen in secondary hyperparathyroidism (B). Answer 3: Increased calcium, decreased phosphate, increased alkaline phosphatase, and increased parathyroid hormone is consistent with serological findings seen in primary hyperparathyroidism (C). Answer 5: Normal calcium, normal phosphate, normal alkaline phosphatase, and normal parathyroid hormone (normal serum findings) are seen in some metabolic bone conditions including osteoporosis and osteopetrosis (E). Bullet Summary: Paget disease of bone is characterized by elevated serum alkaline phosphatase with otherwise normal serum findings.

A 65-year-old man presents to his primary care physician with left hip pain, stiffness, and hearing loss. The patient has a past medical history of diabetes and obesity. He played professional football when he was younger and is currently retired. The patient lives at home alone and admits to having lost some weight recently. His temperature is 99.5°F (37.5°C), blood pressure is 167/98 mmHg, pulse is 90/min, respirations are 17/min, and oxygen saturation is 98% on room air. A CT scan of the head is shown in Figure A, completed as a part of the workup for migraine at a recent emergency department visit. Physical exam reveals bilateral hearing loss but is otherwise unremarkable. Which of the following is the most likely diagnosis? A. Age related hearing loss B. Multiple myeloma C. Osteoarthritis D. Paget disease of the bone E. Rheumatoid arthritis

D, Paget Disease of Bone (Osteitis Deformans) This patient is presenting with hearing loss, bone pain, and a cotton wool appearance on head CT suggesting a diagnosis of Paget disease of the bone. Paget disease of the bone is a condition of abnormal bone remodeling where original osseous tissue is reconstructed through active interplay between excessive bone resorption and abnormal new bone formation. Increased osteoclastic bone resorption is the primary cellular abnormality. These patients are at increased risk of hearing loss, Paget sarcoma, frontal bossing, and cranial nerve palsies. The cause is largely unknown, although it is thought to be related to a viral illness. Figure A shows a CT scan of the head demonstrating a cotton wool appearance with mixed lytic and sclerotic lesions. Diploic widening (both inner and outer calvarial tables are involved) is also apparent. Both of these features are suggestive of Paget disease of the bone. Incorrect Answers: Answer 1: Age related hearing loss does not explain the skeletal abnormalities in this patient. Answer 2: Multiple myeloma presents with bone pain, pathologic fracture, and systemic symptoms (such as fatigue, malaise, and weight loss). Hearing loss, bone pain, and mixed lytic/sclerotic lesions on head CT suggest a diagnosis of Paget disease. While this patient does have some symptoms of multiple myeloma, Paget disease is a more common condition. Answer 3: Osteoarthritis is possible in this obese patient with a history of intense exercise/overuse; however, it would not explain his hearing loss and radiographical findings. Answer 5: Rheumatoid arthritis presents with symmetrical stiffness and joint pain that is worse in the morning and improves with activity. Bullet Summary: Paget disease of the bone presents with bone pain, hearing loss, and mixed lytic/sclerotic lesions on radiography.

A 52-year-old man presents to the emergency department after a motor vehicle collision. The patient is complaining only of left hip pain which has been ongoing for the past 6-8 months. His temperature is 98.5°F (36.9°C), blood pressure is 137/88 mm Hg, pulse is 95/min, respirations are 17/min, and oxygen saturation is 98% on room air. Examination of the left hip reveals mild crepitus, appropriate range of motion, adequate distal sensation, and palpable posterior tibial and dorsalis pedis pulses. A CT scan is ordered as seen in Figure A. Labs demonstrate an alkaline phosphatase of 876 U/L with a normal vitamin D and parathyroid hormone level (PTH). Which of the following is the most likely underlying etiology of this patient's symptoms? A. Multiple myeloma B. Osteoarthritis C. Osteogenic sarcoma D. Paget disease of the bone E. Rheumatoid arthritis

D, Paget Disease of Bone (Osteitis Deformans) This patient is presenting with persistent left hip pain (even prior to the accident), heterogeneous density of the bone noted on CT, and an elevated alkaline phosphatase with a normal calcium/PTH suggesting a diagnosis of Paget disease of the bone. Paget disease of the bone is a metabolic disorder of abnormal bone remodeling. In Paget disease of the bone increased osteoclastic bone resorption and increased osteoblastic bone formation can cause skull deformities and aching bone/joint pain. Patients with Paget disease of the bone will demonstrate a normal calcium and an elevated alkaline phosphatase (from the pathologic bone remodeling). Radiography may demonstrate remodeled cortices and coarsened trabeculae giving the bone a blastic appearance. Other presenting symptoms in Paget disease of the bone include hearing loss (from CN VIII compression), headache, spinal stenosis, radiculopathies, and fractures. The medical treatment of this condition is a bisphosphonate which inhibits the pathologic remodeling. Figure A demonstrates a CT scan of an individual with Paget bone disease. Note the thicker trabeculae and cortex in addition to the heterogeneous density of the bone. Incorrect Answers: Answer 1: Multiple myeloma is a malignancy of monoclonal plasma cells which produce large amounts of Ig. The characteristic finding on radiograph for this pathology is punched-out lytic bone lesions. Patients will often be elderly and present with a pathologic fracture and bony tenderness. Answer 2: Osteoarthritis presents with joint pain and a narrowed joint space on radiography typically in a weight bearing joint with pain worse with exertion and relieved with rest. Answer 3: Osteogenic sarcoma is a malignant neoplasm of the bone that often presents with the classic finding of a radiograph depicting Codman triangle. This triangle is essentially a subperiosteal lesion formed when the periosteum is raised due to the tumor. This is a complication of Paget disease of the bone but is not present on this patient's radiograph. Answer 5: Rheumatoid arthritis typically presents in a woman with symmetric joint pain that is worse in the morning that improves with exertion/by the end of the day. Chronic disease can lead to permanent joint damage and deformities. Bullet Summary: Paget disease of the bone presents with bone pain, an elevated alkaline phosphatase, and sclerotic/lytic lesions on radiography.

An 20-month-old African American female child is brought to the pediatrician by her mother with concerns that the child is not growing normally. The mother recently immigrated from Nigeria. She reports that the child was exclusively breastfed for her first and a half year of life and did not receive any nutritional supplements in the perinatal and postnatal periods. The patient's height is at the 15th percentile for her age. On physical examination, the patient appears moderately malnourished. Prominent thickening of the rib heads at the osteochondral junction is noted. A radiographic image of the patient's legs is shown in Figure A. Which of the following sets of lab findings is most consistent with this patient's condition? A. Normal calcium, normal phosphate, normal alkaline phosphatase, normal parathyroid hormone B. Normal calcium, normal phosphate, elevated alkaline phosphatase, normal parathyroid hormone C. Elevated calcium, decreased phosphate, elevated alkaline phosphatase, elevated parathyroid hormone D. Decreased calcium, decreased phosphate, elevated alkaline phosphatase, elevated parathyroid hormone E. Decreased calcium, increased phosphate, normal alkaline phosphatase, decreased parathyroid hormone

D, Rickets / Osteomalacia The most likely diagnosis in this patient is rickets, a disease caused by decreased vitamin D. Lab findings in rickets include decreased calcium, decreased phosphate, elevated alkaline phosphatase (ALP), and elevated parathyroid hormone (PTH). Rickets is a metabolic disorder characterized by decreased bone matrix mineralization secondary to vitamin D deficiency. Rickets classically presents in children who either have poor dietary intake of vitamin D (e.g., children who are exclusively breast fed without vitamin D supplementation) and/or are not exposed to adequate sunlight. Dark-skinned individuals are at an increased risk of developing vitamin D deficiency because increased skin melanin effectively blocks some UV light. UV light is an important component of vitamin D synthesis in the skin that starts from cholesterol precursors. Signs and symptoms of rickets include short stature, long bone bowing, a waddling gait, thinning of the skull (craniotabes), costochondral thickening of the rib heads (rachitic rosary), sternal protrusion (pectus carinatum), and a depression along the diaphragmatic insertion into the rib cage (Harrison's groove). Because vitamin D levels are low in patients with rickets, absorption of calcium and phosphate in the gut is limited, leading to decreased levels of calcium and phosphate. In response to low calcium, PTH levels rise, which subsequently increases vitamin D production and mineral absorption in the gut. PTH also activates osteoclasts in order to increase serum calcium levels. Osteoclasts break down bone and lead to elevated levels of ALP. Figure A demonstrates the classic leg bowing (genu varum) seen in pediatric rickets. Also note the increased physeal width and cortical thinning. Incorrect Answers: Answer 1: Normal serum findings would not been seen in patients with rickets. These findings are characteristic of healthy individuals but can also been seen in patients with osteoporosis. Answer 2: An isolated elevation in ALP with otherwise normal mineral and PTH levels is characteristic of Paget's bone disease. This condition is associated with localized bone remodeling due to increased osteoclastic reabsorption and abnormal osteoblastic bone formation. Answer 3: Elevated calcium, decreased phosphate, elevated ALP, and elevated PTH are characteristic findings of primary hyperparathyroidism. PTH increases bone resorption, stimulates renal calcium reabsorption, and inhibits renal phosphate reabsorption. The net effect of excess PTH is to increase levels of calcium and alkaline phosphate while decreasing phosphate levels. Answer 5: Decreased calcium, increased phosphate, normal alkaline phosphatase, and decreased PTH are characteristic findings of hypoparathyroidism. In this condition, low PTH leads to reduced bone reabsorption and decreased renal phosphate excretion. The net effect of hypoparathyroidism is to decrease calcium levels and increase phosphate levels. Bullet Summary: Rickets is a condition associated with poor bone matrix mineralization secondary to vitamin D deficiency. Low vitamin D levels lead to hypocalcemia, hypophosphatemia, elevated alkaline phosphatase, and elevated PTH.

A 2-year-old boy presents to the pediatrician for a well-child visit. The child has been doing well and this is his first visit to a pediatrician after being adopted. His parents state that he is doing well and wanted him to generally be checked out. His temperature is 97.0°F (36.1°C), blood pressure is 100/65 mm Hg, pulse is 100/min, respirations are 17/min, and oxygen saturation is 98% on room air. Physical exam is notable for the finding in Figure A. Laboratory values are ordered as seen below. Serum: Ca2+: 9.1 mg/dL Phosphorus: 1.1 mg/dL Alkaline phosphatase: 462 U/L Parathyroid hormone: 23 pg/mL (N = 10-55) 1,25-dihydroxyvitamin D: 22 pmol/L (N = 15-30) What is the most likely diagnosis? A. Pseudohypoparathyroidism B. Renal osteodystrophy C. Vitamin D deficiency rickets D. Vitamin D resistant rickets E. Type II vitamin D dependent rickets

D, Rickets / Osteomalacia This patient is presenting with bilateral bowing of the tibia and femur suggesting inadequate bone mineralization due to metabolic disease. His lab values (normal vitamin D, low phosphorus, elevated alkaline phosphatase, and normal PTH) are consistent with vitamin D resistant rickets (also known as familial hypophosphatemic rickets). Familial hypophosphatemic rickets is the most common form of rickets. It is caused by a X-linked dominant mutation in the PHEX gene leading to an inability of the renal tubules to absorb phosphate (glomerular filtration rate remains normal). This leads to inadequate mineralization of bone and commonly presents as bilateral tibial bowing between the ages of 6 months and 2 years. Laboratory changes include a low serum phosphorous and an elevated alkaline phosphatase. Serum calcium is usually normal (or the low end of normal). High dose vitamin D3 and corrective surgery is currently the standard of care for hypophosphatemic rickets. Figure A is the physical exam finding demonstrating bilateral tibial bowing seen in rickets. Incorrect Answers: Answer 1: Pseudohypoparathryoidism is caused by a defect in the parathyroid receptor making it unresponsive to PTH. These patients will have decreased serum calcium and vitamin D and elevated phosphorous and PTH. Answer 2: Renal osteodystrophy is characterized by the inability to excrete phosphorous as a result of renal failure. As a result of chronic hyperphosphatemia, patients develop hypocalcemia and have increased levels of alkaline phosphatase and PTH. Answer 3: Patients with vitamin D deficient rickets have a nutritional lack of vitamin D and will have decreased serum calcium, phosphorous, and vitamin D in the setting of elevated alkaline phosphatase and PTH. Answer 5: Patients with type II vitamin D dependent rickets have an autosomal recessive mutation leading to a defect in the intracellular receptor for 1,25-dihydroxyvitamin D. They will present with a decreased serum calcium and phosphorous and have an elevated alkaline phosphatase and high levels of vitamin D. Bullet Summary: Vitamin D resistant rickets presents with soft bones (bowing of the lower extremities) and laboratory findings demonstrating normal vitamin D, low phosphorus, elevated alkaline phosphatase, and a normal PTH.

A 12-year-old boy is brought to the clinic for evaluation of right leg pain for the past 2 months. He reports that the pain is concentrated in the right knee area and sometimes radiates towards the thigh. It is especially worse at night and has been waking him up from sleep for the past week or so. His mom reports that the patient has been having trouble climbing up and down the stairs and has been limping while he walks. The patient denies any headaches, night sweats, chills, weight loss, or trauma but endorses subjective fevers and shortness of breath, especially on exertion. A physical examination demonstrates a left-leaning limp with moderate tenderness at the right patella and proximal leg. Both active and passive range of motion are intact, and there is no obvious swelling, erythema, or deformities. A radiograph of the right knee was obtained and is shown in Figure A. What is the most likely explanation for this patient's presentation? A. Autoimmune mediated inflammation and formation of pannus B. Bony exostosis with cartilaginous cap C. Malignant production of osteoid D. Neoplastic proliferation of multinucleated giant cells E. Translocation of chromosomes 11 and 22

E, Ewing sarcoma This patient likely has Ewing sarcoma as demonstrated by the "onion-skin" periosteal reaction seen on the radiograph (figure A), symptoms suggestive of metastasis (e.g., shortness of breath), and presentation of leg pain that is worse at night in the setting of fevers. Ewing sarcoma is associated with a chromosome 11 and 22 translocation leading to fusion protein EWS-FLI 1, which is thought to be connected to the biology of these tumors. Ewing sarcoma is the second most common primary malignancy of the bone (following osteosarcoma). It is more common in males and usually presents in boys between the age of 10-20. Clinical presentation may include swelling and pain, fever, and bone fractures. The most common locations include diaphysis of the femur, tibia, and humerus. Thirty percent of patients may present with metastatic disease on diagnosis, which can include the pelvis, chest wall, and the spinal column. Most of the cases are due to a translocation of chromosomes 11 and 22, which leads to the EWS-FLI1 fusion protein. Diagnosis is based on histomorphology findings, which classically shows small blue round cells. Classic findings on radiogram include lamellated or "onion-skin" type periosteal reactions. Treatment often includes chemotherapy, radiation, surgery, and stem cell transplant. Figure/Illustration A is a radiograph of the right knee demonstrating the classic "onion-skinning" or periosteal reaction (arrow) characteristic of Ewing sarcoma. Incorrect Answers: Answer 1: Autoimmune mediated inflammation and formation of pannus describe rheumatoid arthritis, which would present with pain, swelling, and morning stiffness commonly of small joints of the body (e.g., the wrist, proximal interphalangeal, or metacarpophalangeal joints). In addition, the disease commonly manifests in middle-aged women. Answer 2: Bony exostosis with cartilaginous cap describes osteochondroma, which is a benign bone tumor that can be visualized on the radiograph. This patient's radiograph does not demonstrate this. Answer 3: Malignant production of osteoid describes osteosarcoma, which can present similarly to Ewing sarcoma. However, osteosarcoma commonly involves the metaphysis while Ewing sarcoma commonly appears in the diaphysis of long bones. In addition, this patient has signs of early metastasis (e.g., shortness of breath indicating chest wall involvement), which is more common in patients with Ewing sarcoma. Answer 4: Neoplastic proliferation of multinucleated giant cells describes a giant cell tumor, which is a locally aggressive benign tumor. It often involves the epiphysis of long bones but would look more like "soap bubble" lesions on radiographs. Bullet Summary: Ewing sarcoma is often associated with t(11;22) translocations, and radiographs will demonstrate characteristic periosteal reaction.

A 59-year-old woman presents to her primary care provider complaining of diffuse bodily aches. She reports a 3-month history of gradually worsening pain in her shoulders and hips that is worse in her right hip. She has a history of hypertension and recurrent renal stones for which she takes lisinopril and hydrochlorothiazide. She was admitted to the hospital earlier in the year after falling in her front yard and sustaining a distal radius fracture and vertebral compression fracture. Her temperature is 98.5°F (36.9°C), blood pressure is 145/85 mmHg, pulse is 100/min, and respirations are 20/min. On exam, she is well-appearing with mild tenderness to palpation in her shoulders and hips. She has mild pain with hip flexion and shoulder abduction. She has full range of motion in her bilateral upper and lower extremities. Serum findings are notable for the following: Serum: Na+: 141 mEq/L Cl-: 100 mEq/L K+: 4.8 mEq/L HCO3-: 22 mEq/L Urea nitrogen: 17 mg/dL Glucose: 110 mg/dL Creatinine: 1.12 mg/dL Ca2+: 11.2 mg/dL Phosphate: 2.3 mg/dL Mg2+: 1.9 mg/dL Alkaline phosphatase: 120 U/L A radiograph of this patient's right hip would most likely reveal which of the following? A. Expansile lytic lesion with thin sclerotic margins B. Irregular blastic lesion C. Medullary bone destruction with elevated periosteum from cortical bone D. Poorly marginated lesion extending into adjacent soft tissue E. Well-defined cystic lesion with peritrabecular fibrosis

E, Osteitis Fibrosa Cystica The patient in this vignette presents with bone pain, hypercalcemia, hypophosphatemia, and a history of fragility fractures suggestive of osteitis fibrosa cystica in the setting of primary hyperparathyroidism. Radiographs in patients with osteitis fibrosa cystica may demonstrate brown tumors which appear as well-defined cystic lesions with peritrabecular fibrosis. Osteitis fibrosa cystica is a metabolic bone disorder secondary to severe hyperparathyroidism. In hyperparathyroidism, elevated levels of parathyroid hormone (PTH) increase osteoclast activity, thereby leading to increased bone resorption. This process will lead to the formation of brown tumors, areas of bone loss that are replaced by fibrous bone deposition. Radiographically, brown tumors are characterized as cystic lesions with peritrabecular fibrosis. Incorrect Answers: Answer 1: Expansile lytic lesions with thin sclerotic margins are suggestive of an aneurysmal bone cyst, a benign cystic bone tumor consisting of blood-filled spaces. Answer 2: Irregular blastic lesions may represent metastatic bone lesions from a primary prostate or breast tumor. Answer 3: Medullary bone destruction with elevated periosteum from cortical bone is consistent with the radiographic appearance of osteosarcoma. The periosteal elevation is sometimes referred to as Codman's triangle due to the triangular appearance between the bony cortex and two sides of elevated periosteum. Answer 4: Poorly marginated lesions extending into adjacent soft tissue are consistent with Ewing sarcoma. Ewing sarcoma is a small round blue cell tumor typically occurring in children and young adults. Bullet Summary: Osteitis fibrosa cystica is a manifestation of primary hyperparathyroidism in which increased osteoclastic activity leads to the formation of cystic bone lesions with replacement by proliferating fibrous tissue.

A 70-year-old woman presents to her primary care doctor complaining of left knee pain. She states that she has noticed this more during the past several months after a fall at home. Previously, she was without pain and has no history of trauma to her knees. The patient states that the majority of her pain starts in the afternoon after she has been active for some time, and that the pain resolves with rest and over-the-counter analgesics. Aside from the left knee, she has no other symptoms and no other joint findings. On exam, her temperature is 98.8°F (37.1°C), blood pressure is 124/76 mmHg, pulse is 70/min, and respirations are 12/min. The patient has no limitations in her range of motion and no changes in strength on motor testing. However, there is tenderness along the medial joint line. What finding is most likely seen in this patient? A. Association with HLA-DR4 B. Heberden nodes C. Increased synovial fluid D. Joint pannus E. Marginal sclerosis

E, Osteoarthritis This patient with post-traumatic knee pain which worsens during the day and improves with rest likely has osteoarthritis, which is associated with marginal sclerosis at the joint. Osteoarthritis (OA) is a non-inflammatory arthritis that predominantly affects weight-bearing joints in the body. It is due to chronic mechanical trauma to the articular cartilage of the joint and can be precipitated by trauma to the joint. On plain radiograph, OA is characterized by subchondral cysts, marginal sclerosis at the joint, and osteophytes. Marginal, or subchondral, sclerosis refers to increased bone density seen on plain film at the joint line right below cartilage. There are no systemic symptoms associated with OA, although patients may have other joints affected, including Heberden and Bouchard nodes of the interphalangeal joints. Pain improves with rest, acetaminophen, NSAIDs, and intra-articular steroids. Joint line tenderness can be seen in osteoarthritis but is not specific. Incorrect Answers: Answer 1: Association with HLA-DR4 is seen with rheumatoid arthritis, which has an immunologic basis for disease. Rheumatoid arthritis is more associated with morning stiffness, multiple joints affected, and other systemic symptoms like fever and fatigue. Osteoarthritis is noninflammatory. Answer 2: Heberden nodes are bony growths of the finger distal interphalangeal joints These are associated with osteoarthritis; however, this patient has no other joint findings. Answer 3: Increased synovial fluid can be seen in rheumatoid arthritis. This patient's clinical presentation is more consistent with OA. Answer 4: A joint pannus refers to the proliferation of the synovium in the joint and is seen in rheumatoid arthritis. This patient's clinical presentation is more consistent with OA. Bullet Summary: Osteoarthritis is a non-inflammatory arthritis due to mechanical trauma to the joint and is associated with worsening pain throughout the day that improves with rest and analgesics.

A 64-year-old male presents to his primary care physician with a complaint of bilateral knee pain that has been present for the past several years but has worsened recently. He reports pain with climbing stairs and with extended walks of greater than 100 yards. The pain worsens with activity throughout the day and is alleviated by periods of rest. He states that he has minimal morning stiffness, lasting approximately 5-10 minutes after waking up most days. Physical examination reveals tenderness to palpation of the bony structures on the medial aspect of the bilateral knees as well as crepitus and a decreased range of motion, limited at the extremes of flexion and extension. Both knee joints are cool to touch and exhibit bony enlargement upon palpation of the medial joint line. Which of the following studies would be indicated for further work-up of this patient's presenting condition? A. Complete blood count (CBC) B. Erythrocyte sedimentation rate (ESR) C. Rheumatoid factor (RF) D. MRI of the knee E. No further work-up needed

E, Osteoarthritis This patient is most likely suffering from knee osteoarthritis. Often, the diagnosis of osteoarthritis can be made solely on the clinical history and physical exam findings, without any additional laboratory or radiographic evaluations; however, radiographs may be useful in a preliminary work-up. Osteoarthritis is most commonly seen in patients over 40 years of age, with an increasing prevalence with advancing age. The most common sites of involvement include the hip, knee, vertebrae, and hands. It presents with joint pain, crepitus, and no systemic manifestations. In contrast to a diagnosis of rheumatoid arthritis, symptoms are worsened by continued activity and improved with periods of rest, with minimal morning stiffness lasting less than 30 minutes. Laboratory values, including joint aspiration, are normal; radiographs may show osteophyte formation, joint-space loss, subchondral cysts/sclerosis, or loose bodies in the joint. Hauk reviews the management of knee osteoarthritis. Conservative treatment should include self-management, strengthening exercises, low-impact aerobic exercise, neuromuscular education, and weight loss for patients with BMI > 25. Oral or topical nonsteroidal anti-inflammatory medications or tramadol can be beneficial. Corticosteroid joint injections and surgical intervention are other options if these more conservative measures do not sufficiently alleviate the patient's symptoms. Strand et al. discuss the safety and efficacy of viscosupplementation in the treatment of knee osteoarthritis. A randomized, placebo-controlled trial of almost 5,000 patients showed statistically significant improvements in knee pain and function for patients receiving viscosupplementation over a saline control. There were no differences in safety or adverse effects between the treatment and control groups through 26 weeks. Illustration A summarizes the findings in knee osteoarthritis. Illustration B shows a knee radiograph of a patient with osteoarthritis; note the narrowed joint space, varus deformity, and osteophyte formation. Illustration C is a chart contrasting osteoarthritis with rheumatoid arthritis; note that the wrist, MCP, and PIP joints are more commonly involved in RA versus the PIP and DIP joints in OA. Incorrect Answers: Answer 1: A complete blood count should be normal in a patient with osteoarthritis and would not contribute to making or confirming the diagnosis. Answer 2: ESR is an inflammatory marker that would not be expected to be elevated in cases of osteoarthritis. Answer 3: Rheumatoid factor is elevated in patients with rheumatoid arthritis, which has a differing presentation from that seen in the patient in this vignette. Answer 4: A knee radiograph would be an appropriate test to order to confirm the diagnosis of osteoarthritis; an MRI is rarely ever the first imaging test that should be ordered.

A 55-year-old man presents to his primary care physician for knee pain. The patient has had left knee pain, which has been steadily worsening for the past year. He states that ice and rest has led to minor improvement in his symptoms. He recently bumped his knee; however, he says that it has not altered his baseline pain when ambulating. The patient is a butcher and lives with his wife. His current medications include insulin, metformin, hydrochlorothiazide, and lisinopril. He is attending Alcoholics Anonymous with little success. Physical exam reveals a left knee that is mildly erythematous with some bruising. There is no pain upon palpation of the join or with passive range of motion. The patient exhibits a mildly antalgic gait. Which of the following is the best initial step in management? A. Aspirin B. Colchicine C. MRI D. Rest for 1-2 weeks E. Weight loss

E, Osteoarthritis This patient presents with unilateral joint pain that is chronic and slowly worsening with a mildly antalgic gait, suggesting a diagnosis of osteoarthritis. The best initial step in management is weight loss. Osteoarthritis typically occurs secondary to overuse of a joint and muscular imbalance in a patient who is obese. The pain typically occurs in a cold (non-erythematous and non-edematous) joint with a chronic picture of slowly worsening joint pain. Radiography is an appropriate initial diagnostic test and will show a narrowing of the joint space reflecting a loss of cartilage. The best initial step in management is weight loss and NSAIDs. Incorrect Answers: Answer 1: Aspirin is not an ideal NSAID for osteoarthritis. Ibuprofen would be preferred, although weight loss would be a better initial step. Answer 2: Colchicine could be an appropriate initial step in management for gout which would present with severe sudden pain (typically of the great toe) of a joint with severe pain elicited with passive motion of the joint. Answer 3: MRI could further elucidate the diagnosis; however, radiography would be a better initial step and the clear picture of osteoarthritis warrants intervention rather than further confirmation of the diagnosis. Answer 4: Rest for 1-2 weeks may improve symptoms but will not treat the underlying cause of overuse and obesity. Bullet Summary: The best initial step in management for osteoarthritis is weight loss and NSAIDs.

A 16-year-old boy presents to his primary care physician with left knee pain. He reports a 3-month history of mild progressive chronic knee pain that is not associated with activity. He is an active soccer player and reports that his knee pain is affecting his ability to run. He has a history of mild intermittent asthma and uses an albuterol inhaler as needed. His temperature is 98.6°F (37°C), blood pressure is 115/60 mmHg, pulse is 65/min, and respirations are 16/min. Physical examination reveals a healthy, well-appearing boy in no acute distress. There is small palpable mildly tender mass on the medial inferior left knee. A knee radiograph is shown in Figure A. A biopsy of this patient's condition would most likely reveal which of the following findings? A. Irregular woven bone rimmed by a single layer of osteoblasts B. Large pleomorphic osteoid-producing cells C. Monotonous small round blue cells D. Multinucleated giant cells E. Normal trabeculae with a thin cartilaginous cap

E, Osteochondromas The patient in this vignette presents with a left knee mass arising from the tibial metaphyseal-diaphyseal junction that most likely represents an osteochondroma. Osteochondromas contain normal bone along with a hyaline cartilage cap and would therefore demonstrate normal trabeculae with a thin cartilaginous cap on histologic examination. Osteochondromas (exostoses) are the most common type of benign bone lesion. Though they typically present as painless bone masses, they make become symptomatic if they compress surrounding soft tissues or neurovascular structures. As the name "osteochondroma" suggests, they are composed of both bone and cartilage tissue and resemble a normal physis (growth plate) on histologic examination. Osteochondromas often arise as lateral projections of the physis and will therefore have normal bony trabeculae with a cartilaginous cap. Figure A demonstrates a well corticated pedunculated bony mass growing from the medial proximal tibial metaphyseal-diaphyseal junction and extending distally. No other soft tissue mass, abnormal calcifications, or periosteal reaction is seen. This is likely representative of an osteochondroma. Note that only bony tissue will be seen on a radiograph as cartilage tissue is not visible on radiographs. Incorrect Answers: Answer 1: Irregular woven bone rimmed by a single layer of osteoblasts is characteristic of osteoid osteomas. Osteoid osteomas are small (<1.5 cm) benign bone tumors derived from osteoblasts that are commonly found in the distal femur and proximal tibia. Answer 2: Large pleomorphic osteoid-producing cells are characteristic of osteosarcoma. Osteosarcomas are malignant bone lesions derived from osteoblasts that are commonly found in lone bone metaphyses. Answer 3: Monotonous small round blue cells are characteristic of Ewing sarcoma. Ewing sarcomas are malignant bone lesions derived from neuroectodermal cells that are commonly found in lone bone diaphyses. Answer 4: Multinucleated giant cells are characteristic of giant cell tumors (osteoclastomas). Giant cell tumors are benign but locally aggressive bone lesions derived from osteoclasts that are commonly found in the distal femur and proximal tibia. Bullet Summary: Osteochondromas arise as lateral projections from the growth plate and contain both bone and cartilage tissue.

A 47-year-old African-American woman presents to her primary care physician for a general checkup appointment. She works as a middle school teacher and has a 25 pack-year smoking history. She has a body mass index (BMI) of 22 kg/m^2 and is a vegetarian. Her last menstrual period was 1 week ago. Her current medications include oral contraceptive pills. Which of the following is a risk factor for osteoporosis in this patient? A. Age B. Body mass index C. Estrogen therapy D. Race E. Smoking history

E, Osteoporosis This patient has a significant smoking history which is a risk factor for osteoporosis. Osteoporosis is the most common metabolic bone disease. It is characterized by loss of bone matrix/mineralization, reduced bone mass/density, and reduced thickness of cortical and trabecular bone. Causes of osteoporosis include glucocorticoid use, physical inactivity, hyperthyroidism, and calcium deficiency. Individuals at risk for osteoporosis include Caucasians, women, those with a low body mass index, smokers, and those who are postmenopausal (from decreased estrogen levels). Patients at risk for osteoporosis should be screened using a DEXA scan for central bone density (hip and lumbar spine). The treatment of osteoporosis is elimination of risk factors (such as smoking), weight bearing exercise, and calcium/vitamin D supplementation. Medical treatment for severe or refractory osteoporosis can include bisphosphonates. Incorrect Answer: Answer 1: Age is not a risk factor for this patient as she is not considered elderly and is premenopausal. Given her premenopausal status, smoking is a more important risk factor for her. Answer 2: Body-mass index can be a risk factor if the patient has a low BMI (< 19 kg/m^2). A normal BMI or being overweight is not considered a risk factor. Answer 3: Estrogen therapy is protective against osteoporosis (though this is not an indication for starting oral contraceptive pills or estrogen replacement therapy). Answer 4: Race is a risk factor if the patient is Caucasian; however, African American race is not considered a risk factor. Bullet Summary: Smoking is an important modifiable risk factor for osteoporosis.

An 11-year-old boy with a history of attention deficit disorder presents to a general medicine clinic with leg pain. He is accompanied by his mother. He reports dull, throbbing, diffuse pain in his bilateral lower extremities. He reports that the pain feels deep in his muscles. He has awakened several times at night with the pain, and his symptoms tend to be better during the daylight hours. He denies fatigue, fever, or pain in his joints. On physical examination, his vital signs are stable, and he is afebrile. Physical examination reveals full range of motion in the hip and knee joints without pain. He has no joint effusions, erythema, or warmth. What is the next best step in management? A. Lower extremity venous ultrasound B. MRI of the knees C. Xray of the knees D. Send ESR and CRP E. Reassurance

E, Osteosarcoma Bilateral lower-extremity pain that occurs at night in children from age 2-12 is indicative of growing pain. In the absence of other systemic symptoms, these are normal, and only reassurance is needed. Growing pains are a common occurrence in young children. They are present in 25-40% of children 3-5 and 8-12 years of age. Often they are described as muscular rather than joint pains in the anterior thighs, calves, and behind the knees. Pain often occurs late in the day or at night. Importantly, these children deny swollen, red, warm, or tender joints. If a child appears ill or complains of pain during the day and with activity or if the pain begins to worsen or persist over months, the diagnosis is unlikely to be growing pains. In that case, a thorough evaluation--including CBC and differential, ESR, CRP, and radiographs--is needed. Manners discusses growing pains and other common musculoskeletal problems in children. The typical case of growing pains occurs in a healthy, clinically normal, young child, in the middle of the night, causing intense pain for 10 to 15 minutes in both legs (knees, thighs, calves or shins). Management hinges on simple analgesic measures for the child and reassurance for the parents that a serious condition is not present. Pavone et al. discuss thirty case reports of growing pains. After grouping the patients and conducting a statistical analysis, they found that patients had pain during the night and afternoon in 43.3% and 56.7% of cases, respectively. Both lower limbs were involved in 80% of cases, causing awakening and crying episodes in 40% and 37% of cases, respectively. The frequency of pain was as follows: daily, 5%; weekly, 45%; monthly, 35%; and every 3 months, 15%. The pains were relieved by massaging the affected site in 95% of cases. Incorrect Answers: Answer 1: Lower extremity venous ultrasound would be useful to rule out a deep venous thrombosis, though this diagnosis is less likely in a young patient without risk factors, and bilateral DVTs would be even more rare. Answer 2: MRI of the knees would not be indicated and is better suited to diagnose ligamentous injuries of the knee. Answer 3: A radiograph of the knee would be indicated if the patient were showing signs of joint pathology such a septic knee, but these symptoms are more indicative of growing pains. Answer 4: ESR and CRP may be elevated in a septic joint.

A 16-year-old boy presents to his pediatrician's office after noticing right leg pain for the past 2 weeks. The patient first noticed the pain 2 weeks prior after a soccer game. He did not report any significant injuries at the time. He states that the pain is intermittent, but he tends to notice it while he is in bed at night. Recently, the pain has been getting worse and is a gnawing pain mainly localized to his right lower thigh. On exam, his temperature is 98.2°F (36.8°C), blood pressure is 114/76 mmHg, pulse is 68/min, and respirations are 12/min. There is no gross deformity of his extremities, but he does cite some tenderness to palpation on his anterior thigh immediately superior to his right knee. Motor and sensory function are intact. After imaging is obtained, a bone biopsy is performed and is found to contain pleomorphic cells with osteoid production (Figure A). Which of the following is also associated with this type of lesion? A. Benign exostosis of the bone B. Chromosome fragment translocation C. Normal child development D. Soap-bubble appearance on imaging E. Sunburst pattern on imaging

E, Osteosarcoma This young patient with bone pain and biopsy showing pleomorphic cells with osteoid production likely has an osteosarcoma, which can be associated with a sunburst pattern on radiography. Osteosarcoma is the most common form of primary malignant bone cancer and is found in 2 age groups: patients < 20 and patients > 65. Symptoms often include bone pain that is worse at night and swelling if the tumor is large enough. The tumor is most commonly found at the metaphyses superior to or inferior to the knee joint, but can also be in places such as the pelvis or the jaw. On imaging, the tumor has a "moth-eaten" or "sunburst" pattern from the tumor spicules, but can also produce a Codman triangle when new bone lifts up the periosteum (Illustration A). Histologically, the tumor demonstrates pleomorphic cells producing malignant osteoid (Figure A). Figure A demonstrates poorly-differentiated, pleomorphic cells, with bland, pink-staining osteoid. These are characteristic histologic findings of osteosarcoma. Illustration A demonstrates the Codman triangle indicated with arrows. Bony growth pushing up on periosteum produces this characteristic triangular shape. Incorrect Answers: Answer 1: Benign exostoses of the bone are seen in osteochondromas, which are the most common benign bone tumor. These also affect younger individuals (men under the age of 25). Histology demonstrates mature bone with a cartilage cap. Answer 2: Chromosome fragment translocation is found in Ewing sarcoma, which presents in boys younger than 15. This involves a translocation of chromosome pieces on 11 and 22, which creates a fusion protein. Ewing sarcoma demonstrates small blue cells. Answer 3: Normal child development can be associated with bone pain termed "growing pains." These pains typically occur at night in young children and can often be found behind the knee. Biopsy would be histologically normal. Answer 4: Soap-bubble appearance on imaging is associated with giant cell tumor of the bone. Giant cell tumors are found at the epiphyses of long bones and present in individuals between 20-40 years of age. Histology would demonstrate multinucleated giant cells. Bullet Summary: Osteosarcoma demonstrates a sunburst pattern or Codman triangle on imaging, and pleomorphic cells with osteoid production on histology.

A 70-year-old man presents to the emergency department after a witnessed fall outside of a grocery store. He complains only of left hip pain which has been progressively worsening for the past six months. He denies hitting his head during the fall, but does complains of worsening headaches. His temperature is 98.6°F (37°C), blood pressure is 110/60 mmHg, pulse is 80/min, and respirations are 18/min. Examination of the right lower extremity reveals mild crepitus at the hip, appropriate range of motion, adequate distal sensation, and palpable posterior tibial and dorsalis pedis pulses. Laboratory workup shows the following: Serum: Ca2+: 9.0 mg/dL (normal: 8.9-10.1 mg/dL) Alkaline phosphatase: 876 U/L (normal: 40-120 U/L) 25 hydroxyvitamin D: 89 ng/mL (normal: 20-100 ng/mL) Parathyroid hormone: 30 pg/mL (normal: 10-65 pg/mL) A CT scan of the hips is ordered and shown in Figure A. What is most associated with this patient's condition? A. Osteitis fibrosa cystica B. Osteoblastoma C. Osteochondroma D. Osteoid osteoma E. Osteosarcoma

E,Paget Disease of Bone (Osteitis Deformans) This vignette describes an elderly patient with symptoms and signs concerning for Paget disease of the bone. Osteosarcoma is most associated with this condition. Paget disease of the bone is a progressive metabolic bone disorder of abnormal bone remodeling, characterized by the presence of excessive bone resorption and formation. It is the second most common metabolic bone disorder after osteoporosis. It is most prevalent in individuals > 55 years of age. Increased multinucleated osteoclastic and osteoblastic activity leads to a high bone remodeling rate resulting in abnormal bone architecture with a patchy lytic and blastic pattern observed on imaging. Serum calcium, phosphate, and PTH levels are normal, but alkaline phosphatase is elevated. Untreated Paget lesions can develop into osteosarcoma, an infiltrative and destructive metaphyseal lesion. Figure A shows a CT scan of an individual with Paget disease of the disease. Thick trabeculae and cortex are visible along with heterogeneous density of the bone. Incorrect Answers: Answer 1: Osteitis fibrosa cystica is characterized by decreased bone mass secondary to advanced hyperparathyroidism that gets replaced with fibrosis and brown masses in and around the bone. Answer 2: Osteoblastoma is a neoplasm of osteoid tissue that is greater in size than an osteoid osteoma (> 2 cm). Answer 3: Osteochondroma is a projection of the cartilaginous growth plate that forms an exostosis (new bony growth on surface of bone). It is the most common benign bone tumor. Answer 4: Osteoid osteoma is a benign growth of osteoid tissue that is < 2 cm in size. Bullet Summary: Paget's disease of the bone is associated with osteosarcoma.

A 4-year-old girl falls down on the playground and is rushed to the emergency room. Her work-up reveals a new fracture in her right tibia, as well as multiple old fractures in her bilateral arms and legs at various stages of healing. Her physician is concerned about child abuse, but communication with the girl's pediatrician reaveals her condition. On exam, her sclera are blue.

Osteogenesis Imperfecta / Brittle Bone Disease (snapshot)

A 3-year-old boy presents to the emergency department due to worsening joint pain, fever, and a new rash. He is accompanied by his mother who reports that he has been pointing at his knee and hip while crying. She believes he has been having this symptom for approximately 2 months. He has been having 1-2 fevers a day, and she reports seeing a "pink-like" rash. On physical exam, there is an evanescent salmon-colored rash on the left thigh. There is tenderness to palpation of the left knee and hip with limited range of motion. Lab results show an elevated ESR, CRP, and positive ANA. A pediatric rheumatologist is consulted for further evaluation.

Juvenile Idiopathic Arthritis (snapshot)

A 9-year-old girl presents to his pediatrican's office by his mother due to fever and joint pain. The mother reports that she has been having knee and wrist pain for approximately 7 weeks. Her pain is worse in the morning and is associated with swelling of the affected joint. She develops a fever daily that later subsides. On physical exam, there is tenderness to palpation of the affected joint. Laboratory studies are significant for an elevated ANA, ESR, and CRP. She is negative for rheumatoid factor.

Juvenile Idiopathic Arthritis (snapshot)

A 56-year-old woman presents to her primary care physician with bone pain in her hips, constipation, and anxiety. She reports increased urinary frequency, experiences night sweats, and had an unintentional 15 pound loss over the past 2 months. Laboratory testing is significant for an elevated serum calcium, alkaline phosphatase, and parathyroid hormone and decreased serum phosphate. A computerized tomography (CT) scan of her pelvis is performed.

Osteitis Fibrosa Cystica (snapshot)

A 56-year-old woman presents to her primary care physician with pain in her hands. The pain began approximately 1 year prior to presentation and has progressively worsened. She describes the pain being worse in the evening and improves in the morning. She has also noticed swelling in her knuckles. On physical exam, there is bone deformity noted on the distal and proximal interphalangeal joints, as well as tenderness upon palpation of the affected joints.

Osteoarthritis (snapshot)

A 67-year-old woman presents to the emergency department after falling while walking down the stairs of her home. She landed on her rear on a carpeted floor and denies hitting her head. She experienced severe pain in her right hip after the fall and is unable to bear weight on the affected side. Menopause began 17 years ago. She has smoked 1-pack of cigarettes for the past 40 years. On physical exam, her right leg is shortened, adducted, and externally rotated. Laboratory testing is unremarkable.

Osteoporosis (snapshot)

A 68-year-old woman presents to her primary care physician with lower back pain of acute onset. She denies any trauma to the spine or any radiation of pain. Her last menstrual period was when she was 51-years-old. On physical exam, she has tenderness to palpation at the level of L4-L5, as well as a loss of lumbar lordosis. A dual-energy x-ray absorptiometry (DEXA) scan reveals a T-score of -2.7.

Osteoporosis (snapshot)

An 13-year-old boy presents to his pediatrician for right distal thigh pain. He has been having pain in this area for the past few months, has progressively worsened, and persists in the night. Physical examination is notable for tenderness upon palpation above the right knee. A radiograph of the distal thigh demonstrates a sunburst pattern and Codman triangle.

Osteosarcoma (snapshot)

A 65-year-old man presents with worsening left-sided hearing loss and mild headache. He reported that his symptoms began a few months prior to presentation. He denies starting any new medications but has noticed he can no longer fit the hat he has owned for many years. Physical examination is unremarkable. Laboratory testing is notable for an elevated serum alkaline phosphatase. A radiograph of the skull is shown.

Paget Disease of Bone (snapshot)

A 60-year-old man presents to his primary care physician with hip pain and hearing loss. He denies any trauma to the hip or any recent changes in his medications. He also notes his hat fits tighter than before. Physical examination is unremarkable. Laboratory testing is significant for an elevated alkaline phosphatase level in the setting of normal serum calcium, phosphate, and parathyroid hormone.

Paget Disease of Bone: Osteitis Deformans (snapshot)

A 6-year-old boy is brought to his pediatrician by his parents due to poor growth, weakness, and abnormal gait. On physical exam, there is bowing of the legs and tenderness upon palpation of the lower extremity. Laboratory testing is significant for a decreased serum calcium and phosphate levels and elevated parathyroid hormone and serum alkaline phosphatase levels.

Rickets secondary to vitamin D deficiency (snapshot)

A 7-year-old girl is brought to the emergency department after experiencing a risk fracture. The patient fell down to the ground and has not hit her head. This has never happened before. The patient's parents reports their child states she has pain in bones and at times feels weak. On physical exam, there is tenderness to palpation of the wrist with a waddling gait. Laboratory testing is notable for a decreased serum calcium and phosphate level, and increased alkaline phosphatase and parathyroid hormone.

Rickets secondary to vitamin D defiency (snapshot)


संबंधित स्टडी सेट्स

Colby College PS111 Final Exam study guide

View Set

12th Grade Government Midterm Review

View Set

Week 7 | PrepU | Chapter 09 - ML 4 | Teaching and Counseling

View Set

8.1: Electric potential energy and voltage

View Set

6.14) Types of Heuristics: Availability, Representativeness & Base-Rate

View Set